You are on page 1of 60

Pathophysiology 2019

Set 1 checked
1. A 49-year-old man was suffering 12 years ago from rheumatic myocarditis, endocarditis, and
insufficiency of mitral valve. Examinations showed the absence of inflammatory process,
sufficient minute blood volume. What is it?
A. Pathological reaction
B. Pathological process
C. Typical pathological process
D. Compensatory reaction
E. Pathological condition
2. The 12-year-old boy took part in sport events in light athletic. Weakness, headache, loss of
appetite, and rise in temperature to 37,8oC appeared in him on the next day. What period of
disease is it?
A. Final period of disease
B. Period of manifestation
C. Contact period
D. Latent period
E. Prodromal period
3. Pain in a left half of the chest and dyspnea appear in a patient during walking. This pain
increases during respiration. The attack of the pain was stopped by using narcotics. At
examination of the patient following data were revealed: severe condition, respiratory rate is 28
per min., respiration is shallow and is dull in left axillary region. Blood pressure is 140/80
mmHg. Patient has sputum with blood. This patient has varicose dilation of veins. In purpose to
correct main pathogenetic link it is necessary to use:
A. Antibiotics
B. Anticoagulants
C. Antihistamin preparations
D. Spasmolytics
E. Coronarilytics
4. A 39-years-old patient has been suffering from gastric ulcer for last 4 years. Pain in epigastric
region, heartburn, nausea, and constipation appear mainly in autumn and spring. Name this
condition.
A. Remission
B. Acute period
C. Complication
D. Pathologic condition
E. Relapse
5. Gasping respiration appears in a patient with severe lung pathology. What terminal condition
is this characteristic for?
A. Agony
B. Pre-agony
C. Clinical death
D. Biological death
E. Terminal pause
6. A 10-yeaes-old child endured several rheumatic attacks. At examination of him it was
established that he had inflammatory process in his joints and signs of mitral valve insufficiency.
What pathological appearance in this patient may be attributed to “disease”?
A. Mitral valve disease
B. Mitral valve insufficiency
C. Rheumatism
D. Inflammation of joints
1
E. Arthritis
7. Which scientist emphasized senescence of connective tissue cells cytoplasm?
A. Bogomolets
B. Mechnikov
C. Dilman
D. Frolkis
E. Berdichev
8. A 12-year-old boy starts complaining on the headache, nausea, rigor, periodic myalgia, loss of
appetite, tiresome. Which period of disease characterizes by such symptoms?
A. Prodromal period
B. Latent period
C. Period of contact
D. Period of manifestation
E. Final period
9. A 28-year-old patient with symptoms of acute appendicitis was admitted to the surgical
department. Acute pains in his right epigastric area and near the umbilicus were registered during
palpation on examination. Schetkin-Blumberg symptom was positive. Which period of disease
was observed in that patient?
A. Period of manifestation
B. Latent period
C. Prodromal period
D. Final period
E. Period of functional disorder
10. Increase in pulse rate, respiratory rate, and increase of BP were noticed on a height of 1000
m above the sea in a 25-year-old woman, coming for holyday at mountainous resort. In some
time all those symptoms disappeared. Which process was noticed in a woman?
A. Adaptation
B. Compensation
C. Decompensation
D. Stress
E. Parabiosis
11. In a 50-year-old man, who was treated for gastric ulcer, digestion normalized, pains
disappeared, and mood is getting better. But in few weeks pains in epigastrium, heartburn, and
acid eructation develop again. How can one characterize such course of disease?
A. Relapse
B. Remission
C. Terminal condition
D. Typical pathological process
E. Latent period
12. A man with the complex of deviation of his health is considered sick. What is the more
typically for disease?
A. Disorder of physiological regulation of functions
B. Decrease of ability to work
C. Disorder of immunity
D. Psychological deviations
E. Decrease of adaptation

Set 2 checked
1. A worker of radiological department was exposed to radiation once as result of violation of the
rules of safety appliances. Ulcerogangrenous stomatitis developed in him in 8 days. Patient’s
blood test showed: RBC – 3.2 x 1012/L, reticulocytes – 0.01 %, Hb – 60 g/L, WBC – 2.3 x
2
109/L, and platelets – 50 x 109/L. Which period of radiation sickness are described changes
typical for?
A. Period of primary reactions
B. Period of manifestation
C. Latent periods
D. Pretended well-being period
E. Outcomes
2. Respiratory standstill developed in a man as a result of action of electric current from town
mains for 0.1 seconds with position of electrodes “right hand – head”. Indicate the most probable
reason for this complication.
A. Paralysis of inspiration center
B. Emotional stress
C. Paralysis of respiratory muscles
D. Reflex respiratory standstill (pain shock)
E. Total paralysis of respiratory center
3. Increase of body temperature up to 38.5oC, tachycardia, and BP of 160/85 mmHg were
noticed in a worker after prolonged work in conditions of “warming” microclimate. Disturbances
of heart rhythm were revealed in the patient at auscultation. Which electrolyte primary loss
affects adversely the heart work?
A. Potassium
B. Calcium
C. Magnesium
D. Sodium
E. Phosphate
4. Elevation of arterial pressure, tachycardia, and increase in body temperature to 38,8oC
developed in a patient during narcosis with myorelaxants. Which disturbance of heat exchange
appeared in this patient?
A. Fever
B. Physical hyperemia
C. Hit heat
D. Hypothermia
E. Hyperthermia
5. At violation of the rules of safety appliances an electric welder took electric wire by
unprotected and wet hand. Tonic contractions of muscles and involuntary urination developed in
him. Which effect of electric current becomes apparent in this case?
A. Electrochemical
B. Biological
C. Heat
D. Mechanical
E. --
6. During long hypothermia of the organism phase of decompensation developed. Its feature is:
A. Decrease in blood pressure
B. Constriction of peripheral vessels
C. Rare respiration
D. Increase in muscle tone
E. Rigor
7. A man took electric wire with high tension by both hands. He died momentary in result of:
A. Intracerebral bleeding
B. Respiratory standstill
C. Cardiac fibrillation
D. Burns
E. Tearing extremities off
3
8. The main link in pathogenesis of kinetosis under radial acceleration is:
A. Excitation of motor nuclei of vagus nerve
B. Irritation of tactile receptors
C. Increase in muscular tone
D. Irritation of nuclei of vestibular nerve
E. Bradycardia
9. A worker, taking part in nuclear power plant breakdown elimination, was exposed to whole-
body irradiation. Which organ or which tissue will tumor develop in?
A. Lung
B. Bone marrow
C. Stomach
D. Skin
E. Bone tissue
10. A worker, taking part in nuclear power plant breakdown elimination, was exposed to ionizing
radiation in dose of 6Gr. What changes in the blood will happen after 10 days?
A. Agranulocytosis
B. Limpocytosis
C. Leucocytosis and limphocytopenia
D. Basophilia
E. Eosiniphilia
11. A man had electric trauma. Current went through the myocardium. What dangerous disorders
in heart work needing emergency correction can appear in such situation?
A. Fibrillation of ventricles
B. Fibrillation of atria
C. Bradycardia
D. Tachycardia
E. Extrasystolia
12. A worker was exposed to ionizing radiation in dose of 5Gr after accident at nuclear power
plant. Agranulocytosis was revealed in him in a week. What is the basic pathogenic mechanism
in this case?
A. Increased outcome of mature leukocytes from bone marrow
B. Development of autoimmune processes
C. Increase in migration of granulocytes to tissues
D. Elevated destruction of leukocytes
E. Oppression of leucopoiesis
13. A patient has bone marrow form of radiation diseases. What lineages of hemopoiesis are
affected in this case?
A. All
B. Erythrocytic
C. Megakariocytic
D. Granulocytic
E. Agranulocytic
14. Symptoms of hemorrhage syndrome appeared in a patient with radiation sickness. Which
mechanism is leading one in pathogenesis of this syndrome?
A. Thrombocytopenia
B. Lymphopenia
C. Erythropenia
D. Eosinopenia
E. Neutropenia
15. A doctor revealed in a patient affected by electric current respiratory arrest and disturbances
of heart functions. What pathway did electric current go in this case?
A. Right hand - head
4
B. Left leg - right leg
C. Right leg - right hand
D. Left leg - right hand
E. Abdomen - right leg
16. Diver, working at depth of 60 m, was raised very quickly on surface after an accident. After a
time pain in the joints, skin itching appeared in him. What happened to diver?
A. Decompression diseases
B. High altitude diseases
C. Hyperbaric oxygenation
D. Lowering of reactivity
E. Compensatory adaptation
17. A galvanizer addressed to a doctor because of appearing of pink itching spots and vesicles on
his arms skin. He worked a lot with nickel compounds recently. What is the mechanism of these
lesions development?
A. Modification of skin proteins due to action of nickel ions with immunopathological reaction
development
B. Irritant action of nickel compounds on skin
C. Vegetative disorders related to nickel compounds received by respiratory tract
D. Infection of skin due to microtraumas
E. Nickel is a complete antigen
18. In consequence of accident in laboratory man was exposed to radiation in dose of 6Gr.
Which symptom is the most typical for latent period of acute radiation diseases?
A. Leukocytopenia
B. Headache
C. Depression
D. Vomiting
E. Diarrhea
19. A patient was admitted to the hospital with extensive burns of the body surface. Which
pathogenic factor of burn disease has to be liquidated in the first turn?
A. Intoxication of decay products
B. Plasma loss
C. Pain
D. Infection through the injured skin
E. Autoallergy
20. During running parallel experiments rats were exposed to prolonged direct solar radiation in
open chambers and in ones covered by glass. Tumor development at hair uncovered places of
skin was marked in animals residing in open chambers. Which factor influence does this
phenomena connected to?
A. Sun heat
B. Ultraviolet radiation
C. Biologic carcinogens
D. Exogenous chemical carcinogens
E. Infrared radiation
21. In consequence of accident at nuclear power plant radio-active products escape occurred.
Peoples, who were in zone of increased radiation, got dose of radiation about 250 – 300 Gr.
What symptom would be the leading one in these people?
A. Lymphocytosis
B. Anemia
C. Thrombocytopenia
D. Neutropenia
E. Leukocytosis

5
22. A man was admitted to the hospital after exposure to radiation in dose of 3Gr. Which
physiological system develops changes in the first turn in this patient?
A. Blood system
B. Cardiovascular system
C. Immune system
D. Alimentary system
E. Endocrine system
23. Headache, pain in ears, nasal hemorrhages appeared in the passengers during flight by
balloon. What serves the pathogenic base for these changes development?
A. Explosive decompression syndrome
B. Decompression syndrome
C. Hypoxia
D. Gas embolism
E. Toxic influence of gases dissolved in the blood
24. What is the reason for pneumonia development in children after cooling?
A. Microorganisms
B. General cooling
C. Decrease in reactivity due to previous diseases
D. Insufficiency of nutrition
E. All these factors
25. Which changes are connected to direct injurious action of ionizing radiation?
A. Inhibition of DNA-synthesis under the influence of radiotoxines
B. Radiolysis of water
C. Edema
D. Interaction between free radicals and enzymes
E. Injury of chromosomes
26. What reasons does high radiosensitivity of cells and tissues depend on?
A. High mitotic activity
B. Activation of glycolysis
C. Level of differentiation
D. Number of mitochondrias
E. Decrease of DNA-synthesis
27. Which changes in blood are typical for the first period of bone marrow form of acute
radiation diseases?
A. Leukocytosis
B. Agranulocytosis
C. Thrombocytopenia
D. Erithrocytopenia
E. Leukocytopenia
28. Among many factors determining severity of electrical injury the first place belongs to injury
of internal organs localized at the way which electric current passes through the body. Which
organ injury is the most dangerous?
A. Heart
B. Cerebrum
C. Lungs
D. Liver
E. Kidneys
29. A patient with III – IV degree burns of 50% of the body surface was admitted to the hospital.
He has depressed consciousness, bradycardia, and reduced blood pressure. What is the most
possible diagnosis?
A. Burn shock
B. Intoxication
6
C. Preagony
D. Collapse
E. Coma
30. A patient very often has diseases of nasopharynx, which appear under the influence of
different factors and in the most cases manifest by inflammation. Which of listed factors is the
most probable reason for these diseases?
A. Microorganisms
B. General cooling of organism
C. Overstrain
D. Immunodeficiency
E. Insufficiency of nutrition
31. A woman, who took part in liquidation of accident at Chernobyl nuclear power plant, was
admitted to the hospital with complaints of headache, pain in the heart, and worsening of general
condition. Changes of subcellular structures were found out in her, except:
A. Proteins
B. Nucleus
C. Lisosomes
D. Mitochondrion
E. Endoplasmic reticulum
32. A 45-year-old man suffering from hematoporphiria complains of burns of skin, appeared
after short-time (no more than 10 to 20 minutes) exposure to the sun, headache, nausea, and
dizziness. Explain the reason for this pathology?
A. Photoallergy
B. Sun stroke
C. Heat stroke
D. Photochemical burn
E. Hyperthermia
33. A liquidator of accident at Chernobyl nuclear power plant after carrying out works addressed
to a doctor with complaints of abrupt worsening of general state, significant weakness, increase
in body temperature up to 37,8oC, multiple punctate hemorrhages on skin and mucous
membranes, diarrhea. In the patient’s blood analysis ESR – 25 mm/h, RBC – 2,4x1012/L, WBC
- 2,2x109/L, platelets – 70x109/L. What period of radiation disease does this picture typical for?
A. Manifestation
B. First reaction
C. Pretended well-being period
D. Recovery

Set 3 checked
1. Enzyme trypsin is released by the pancreas as a non-active proenzyme trypsinogen. What is
the mechanism of proenzyme trypsinogen activation?
A. Allosteric activation
B. Elevation of de novo synthesis at genetic level
C. Change of quartiary structure of molecule
D. Phoshporilization of molecule
E. Restricted proteolysis

2. Necrosis of cells develops after they were exposed to high dose of ultraviolet irradiation. What
reactions underlie these changes?
7
A. Hydrolysis of proteins
B. Hydrolysis of lipids
C. Decarboxylation of amino acids
D. Peroxidation of lipids
E. Oxidation of carbohydrates
3. Increase in levels of Alanine Aminotransferase (AlAT) and Aspartate Aminotransferase
(AsAT) was found during examination of patient’s blood serum. What changes in organism at
cellular level lead to this situation?
A. Disturbances of intercellular interactions
B. Disorders of enzyme systems of cells
C. Impairment of energy supply of cells
D. Destruction of cells
E. Damage of genetic apparatus of cells
4. Increase in activity of Lactate Dehydrogenase was found during examination of patient’s
blood serum. What changes at cellular level lead to such disturbances?
A. Damage of plasmatic membranes
B. Disorders of energy supply
C. Disturbances of intercellular interactions
D. Damage of genetic apparatus of cells
E. Inhibition of enzyme systems
5. Cell of an experimental animal was exposed to X-rays. Protein fragments were formed in
cytoplasm as a result. What cellular organelles participate in their utilization?
A. Lysosomes
B. Glogi apparatus
C. Ribosomes
D. Endoplasmatic reticulum
E. Cell center
6. Activity of Alanine Aminotransferase (AlAT) and Aspartate Aminotransferase (AsAT)
sharply increases in blood plasma at hepatitis and myocardial infarction. What are the reasons for
increase in their activity in blood?
A. Damage of cell membranes and release of enzymes to the blood
B. Augmentation of enzyme activity by hormones
C. Pyridoxine deficiency
D. Increase in rate of amino acids synthesis in tissues
E. Increase in rate of amino acids decay in tissues
7. Tissue culture was exposed to radiation. Nucleoli in cell nuclei were damaged. What
organelles resumption becomes problematic in cell cytoplasm?
A. Endoplasmatic reticulum
B. Ribosomes
C. Lysosomes
D. Golgi apparatus
E. Microtubules
8. Activation of universal membrane structure injuring mechanism occurs at reperfusion
syndrome. This mechanism is referred to as
A. Beta-oxidation of lipids
B. Oxidation of cytochromes
C. Microsomal oxidation
D. Knoop-Lienen cycle
E. Peroxidation of lipids
9. It was established that pneumonia was caused by viruses in a 5-year-old child. What is the
leading mechanism of cell injury in this pathology?
A. Protein
8
B. Electrolyte-osmotic
C. Nucleic
D. Lipid
E. Calcic
10. Destruction of cartilage cells and disorder of their functions are observed in a patient with
rheumatism. Which of cellular organelles takes active part in this pathological process?
A. Ribosomes
B. Microtubules
C. Lysosomes
D. Golgi apparatus
E. Cellular center
11. Which ions accumulation in cytoplasm of muscular cells accounts for stable constriction of
myofibrils?
A. Calcium
B. Sodium
C. Potassium
D. Magnesium
E. Hydrogen
12. What is the direct reason for disorders of calcium-ions extraction from cytoplasm during cell
injury?
A. ATP deficiency
B. Acidosis
C. Increase of osmotic pressure in cytoplasma
D. Increase of permeability of cell membranes
E. Denaturation of proteins
13. At the phase, preceding diastolic relaxation of myocardium, concentration of calcium ions
sharply increases in sarcoplasm, and uncombined calcium is virtually absent from sarcoplasm at
diastole. Which of following structures participates in accumulation of calcium?
A. Mitochondrions
B. Lysosomes
C. Ribosomes
D. Golgi apparatus
E. Nucleoli
14. During study of cell structure by students the question appears “What are biological
membranes by their structure?”
A. Bimolecular lipid layer with protein constituents
B. Bimolecular lipid layer
C. Monomolecular lipid layer
D. Bimolecular protein layer
E. Bimolecular protein layer with lipid constituents

Set 4 checked
1. There are no reactions of delayed type hypersensitivity in mice without hairs (nude mice). The
most possible reason for this pathology is:
A. Defect of phagocytosis
B. Disorders of hematopoiesis
C. Absence of thymus
D. Deficiency of components of complement systemsss
E. Absence of gamma globulins in blood
9
2. Transplantation of skin was performed in a patient with wide spread burns. Graft swelled and
changed its color at 8th day and was rejected at 11th. What cells participate in this process?
A. Erythrocytes
B. Basophils
C. Eosinophils
D. B-lymphocytes
E. T-lymphocytes
3. Deficient content of immunoglobulins was revealed in a patient. What cells of immune system
produce immmunoglobulins?
A. Plasma cells
B. T-killers
C. B-lymphocytes
D. T-helpers
E. T-suppressors
4. Three times immunization of population was carried out by pertussis-diphtheriatetanus
vaccine when the number of diphtheria cases rose. The levels of anybodies increased in the
blood of immunized people as a consequence of immunization. What cells produce these
proteins?
A. Acidophilic granulocytes
B. Neutrophilic granulocytes
C. Macrophages
D. Monocytes
E. Plasmocytes
5. It is known that plasma cells produce specific antibodies against the given antigen. Number of
plasma cells increases after introduction of antigen. What cells of peripheral blood serve as
precursors of plasma cells?
A. Neutrophils
B. B-lymphocytes
C. Basophils
D. T-lymphocytes
E. Eosinophils
6. Formation of T-helpers is held up in thymus. What processes of immunogenesis in connective
tissue will be violated at first?
A. Phagocytosis of antigens by macrophages
B. Opsonization
C. Conversion of B-lymphocytes to plasma cells
D. Phagocytosis of foreign entities
E. Formation of precursors of T-lymphocytes
7. Transplantation of donor heart was performed in a patient. What conditions have to be
observed to prevent transplant rejection?
A. Selection of donor according to HLA
B. Transplantation of bone marrow
C. Transfusion of donor’s blood
D. Removal of spleen
E. Administration of immunomodulators
8. A patient was operated for acute purulent appendicitis. He cannot be discharged from the
hospital for a long time because of bed healing of post-operative wound. He has diabetes
mellitus for many years, repeated pyoderma, furunculosis, stomatitis, and gingivitis. What is the
reason for decreased immunologic reactivity?
A. Hyperglycemia
B. Hypercholesterolemia
C. Hyperketonemia
10
D. Hypohydration
E. Protein metabolism violation
9. It is known that reactivity of the organism is opposite to its resistance during some
pathological processes. What pathological process can appear in such situation?
A. Fever
B. Shock
C. Inflammation
D. Posthemorrhage anemia
E. Arterial hypertension
10. Macrophages surrounded with alien erythrocytes were found at microscopic examination of
smear of exudation took from a rat suffering from aseptic peritonitis with addition of bird
erythrocytes. What stage of phagocytosis does this occurrence correspond to?
A. Incomplete phagocytosis
B. Adhesion
C. Hemotaxis
D. Engulfment
E. Intracellular digestion
11. A 3-year-old boy suffering from chronic pneumonia has low indices of B-lymphocyte
system. Bruton’s hypogammaglobulinemia was diagnosed in him. What is the consequence of B-
lymphocyte system deficiency?
A. Decreased resistance of organism to pyogenic coccus flora
B. Decreased resistance of organism to viruses
C. Decreased resistance of organism to fungous and tuberculosis infection
D. Absence of graft rejection reactions
F. Increased risk of tumor development in organism
12. Considerable edema of lips appeared in a 25-year-old man in dentist’s office few minutes
later washing his mouth with solution of furacillin. Which type of allergic reaction is observed in
this case?
A. Immune complex-mediated
B. Delayed-type hypersensitivity
C. Anaphylactic
D. Stimulating
E. Cytotoxic
13. Child has congenital heart disease, face defect, absence of thyroid gland and thymus, and T-
lymphocytes in blood. What hereditary pathology are these symptoms connected with?
A. Di George syndrome
B. Luis-Barr syndrome
C. Turner’s syndrome
D. Down’s disease
F. Bruton’s disease
14. A patient with thyrotoxicosis has antithyroid antibodies in his blood. Which type of allergic
reactions is observed in this case?
A. Cytotoxic
B. Stimulating
C. Delayed-type hypersensitivity
D. Anaphylactic
E. Immune complex-mediated
15. A 2-month-old girl with pneumonia was admitted to pediatric department. She bore otitis and
pneumonia a month ago. Dicrease in levels of gamma-globulins by 2 times was found at her
examination. Which disorder of immune system is it?
A. Luis-Barr syndrome
B. Physiological hypogammaglobulinemia
11
C. Late hypogammaglobulinemia
D. Swiss type of immune deficiency
E. Di George syndrome
16. A 2-year-old boy has frequent bacterial infections since 2 months. Allergic reaction to
tuberculin (IV type) is positive. Which immunodeficiency is more possible at this patient?
A. Congenital B-lymphocytes insufficiency
B. Congenital T-lymphocytes insufficiency
C. Congenital T-suppressors defect
D. Congenital total immunodeficiency
E. Acquired immunodeficiency
17. Association of staphylococcus aureus and staphylococcus epidermidis are found in numerous
skin pustules at 25-year-old patient. Pneumocysta carinia is found in his sputum analysis.
Cryptosporidia, proteus vulgaris and fungi of candida genus are found in his stool. Which
disease is accompanied by such multiple infections with conditionally-pathogenic
microorganisms?
A. Dysbacteriosis
B. AIDS
C. Sepsis
D. Diabetes mellitus
E. Drag agranulocytosis
18. A 25-year-old man complains of frequent inflammation diseases of different localization. He
is injection drug addict. HIV-test was positive at him. What types of cells are damaged more
substantially in case of HIV?
A. Neutrophilic granulocytes
B. Plasmocytes
C. Macrofages
D. T-helpers
E. NK-cells
19. A 12-year-old boy with hereditary disease has eczema, 4 cases of pneumonia in his
anamnesis, bleedings. Low level of T-lymphocytes, decreased levels of IgM, normal levels of
IgA was found in his blood. What disease does the boy have?
A. Wiskott-Aldrich syndrome
B. Luis-Barr syndrome
C. Bruton’s disease
D. Late hypogammaglobulinemia
E. Early hypogammaglobulinemia
20. A newborn has recurring respiratory tract diseases, pustule affections of the skin, severe
clinical course of herpes infection and recurring candidiasis (thrush). Inoculation against
smallpox and BCG-vaccination was accompanied by severe complication in him. All classes of
immunoglobulins were decreased in his immunological test. Delayed-type hypersensitivity
reactions were absent in him. What cells insufficiency underlies this immunodeficiency
syndrome?
A. Neutrophiles
B. T- and B-lymphocytes
C. Macrophages
D. Fibroblasts
E. T-suppressors
21. Father has rhesus-positive erythrocytes and mother’s blood is rhesus-negative. Severe
hemolytic anemia has developed just after birth in their second child. What is the mechanism of
massive hemolysis at newborn’s organism?
A. Toxemia of pregnancy
B. Antenatal intoxication
12
C. Antenatal infection
D. Genetic deficit of erythrocytes
E. Immune hemolysis
22. A 1-year-old boy becomes very often ill with respiratory and pustular skin diseases. Even
little damages become complicated by long-time purulent inflammation. All classes of
immunoglobulins are nearly absent from the child’s blood. Contents of all leukocyte types and
reactions of delayed-type hypersensitivity are normal. What cells inherited decrease in functional
activity underlies this syndrome?
A. T-lymphocytes
B. B-lymphocytes
C. Neutrophils
D. Eosinophils
E. Endoteliocytes
23. Decrease in levels of IgG and particularly IgA, IgM was found during examination of
immune status in 5-year-old boy. B-lymphocytes and plasma cells are absent from his blood and
lymph nodes. Reactions of T-lymphocytes are normal. This is inherited sex-linked disease. What
diagnosis is more possible?
A. Bruton’s disease
B. Luis-Barr syndrome
C. Wiskott-Aldrich syndrome
D. Swiss type of immunodeficiency
E. Early hypogammaglobulinemia
24. The reasons for disturbances of phagocytosis may be qualitative disorders of phagocytes.
What changes of intracellular structures are more typical for “lazy leucocytes” syndrome?
A. Chemotaxin and opsonin receptors
B. Microfilaments
C. Specific membrane glycoprotein (GP 110)
D. Microtubules
E. Bactericidal system
25. A 9-year-old girl was getting breast feeding during the first year of life. She suffered
prolonged severe pneumonia in the end of the first year. She started walking late. Her gait was
unsteady and her movements were discoordinated. Telangiectasia appeared in her skin and
conjunctivas. IgA is absent from and levels of T-lymphocytes is decreased in her blood. What
immunodeficiency disease does the girl have?
A. Luis-Barr syndrome
B. Wiskott-Aldrich syndrome
C. Chediak-Higashi syndrome
D. Swiss type of immunodeficiency
E. Di George syndrome
26. Immune system is depressed in a patient with HIV-infection. What cells damage contributes
more to immunodeficiency at this patient?
A. T-suppressors
B. Macrophages
C. T-helpers
D. B-lymphocytes
E. T-killers
27. A 2-year-old patient suffering from chronic pneumonia has decrease in quantity of B-
lymphocytes, IgM and IgG. What microorganisms can cause development of this disease?
A. Candidas
B. Mycobacterium tuberculosis
C. Herpes virus
D. Adenovirus
13
E. Staphylococcus
28. Disorder of cellular immunity was revealed during investigation of immune system in the
patient with chronic fungus affection of the skin. What indexes decrease more typically in this
case?
A. IgG
B. IgE
C. T-lymphocytes
D. B-lymphocytes
E. Plasma cells
29. Mantoux test was made to sick child on suspicion of tuberculosis. Swelling, hyperemia and
painfulness occur in the allergen injection place in 24 hours. What components determinate this
reaction of organism?
A. Mononuclear cells, T-lymphocytes and lymphokines
B. B-lymphocytes and IgM
C. Plasma cells, T-lymphocytes and lymphokines
D. Granulocytes, T-lymphocytes and IgG
E. Macrophages, B-lymphocytes and monocytes
30. A patient was suffering from chronic renal insufficiency. He has renal transplantation from
donor, who died in motorcar accident. Donor had blood group same as recipient. Renal
insufficiency occurs in patient again after a time. Which system incompatibility takes part in
transplant rejection?
A. ABO
B. RH
C. MNS
D. HLA
E. Kel-Chelano
31. HIV-infection was suspected at a patient of dental clinics. What cells are damaged at first in
patient with AIDS?
A. T-helpers
B. T-suppressors
C. T-killers
D. B-lymphocytes
E. Macrophages
32. A 48-year-old patient suffers from frequent mycotic lesions and pyoderma at surface of trunk
for a long time. He has alcohol abuse in his anamnesis. What promotes realization of disease
reason in this case?
A. Damage of the blood-brain barrier
B. Primary immunodeficiency
C. Decreased barrier function of the liver
D. Allergy
E. Chronic pneumonia
33. A 10-year-old girl suffers from viral and mycotic diseases very often. She has congenital
heart disease and insufficiency of thyroid gland. T-lymphocytes is absent from her
immunological test. What disorder of immune system takes place in this case?
A. Bruton’s hypogammaglobulinemia
B. Mixed immunodeficiency
C. Terner’s syndrome
D. Hypoplasia of thymus
E. Inherited defect of complement system
34. Pneumonias and pyodermas very often occur in 5-year-old boy, as well as in his grandfather.
B-lymphocytes are absent from their immunological test. What disorder of immune system takes
place in this case?
14
A. Bruton’s hypogammaglobulinemia
B. Hypoplasia of thymus
C. Mixed immunodeficiency
D. Terner’s syndrome
E. Inherited defect of complement system
35. Viral and bacterial infections, eczema often occur in a 12-year-old boy. Decreased quantity
of T-lymphocytes and IgM and normal levels of IgG and IgA are found in him at examination.
What kind of immune pathology is observed in this case?
A. Hypoplasia of thymus
B. Mixed immunodeficiency
C. Bruton’s hypogammaglobulinemia
D. Terner’s syndrome
E. Inherited defect of complement system
36. A 20-year-old woman with intestinal polyposis has frequent mycotic and viral diseases in her
anamnesis. What link of immune system is insufficient more possibly in this case?
A. T-lymphocytes
B. B-lymphocytes
C. Natural killers
D. Complement
E. Phagocytosis
37. Which of following is immunodeficiency disease?
A. Inherited disorder of immune system
B. Decrease of immune reactions under avitaminosis
C. Insufficiency of immune reactions after exposure to ionizing radiation
D. Disorder of formation antibodies as consequences of protein starvation
E. Decrease in immune reactions under cytostatic drugs action
38. A Patient needs renal transplantation. What kind of transplantation more expedient?
A. Syngenic
B. Xenogenic
C. Autogenic
D. Allogenic
E. Explantation
39. Heart transplantation is indicated for the patient. What kind of transplantation more
expedient convenient)?
A. Syngenic
B. Xenogenic
C. Autogenic
D. Allogenic
E. Explantation
40. The main role in pathogenesis of diseases, which occur in case of B-lymphocytes
immunodeficiency, plays:
A. Immunoglobulin synthesis disorders
B. Cellular type immune reaction disorders
C. Loss of ability to graft rejection
D. Decrease in delayed-type hypersensitivity
E. Decrease in antitumor immunity
41. A newborn has convulsive syndrome and heart interventricular septum defect. Thymic
hypoplasia was revealed by X-ray examination of his chest. What immunodeficiency may be
supposed in this patient?
A. Di George syndrome
B. Bruton’s disease
C. Wiskott-Aldrich syndrome
15
D. Good’s syndrome
E. Luis-Barr syndrome
42. A 15-year-old patient was admitted to allergologic department with bronchial asthma. What
immunoglobulins excessive production can result in main clinical symptoms development in this
patient?
A. IgE
B. IgA
C. IgG
D. IgM
E. IgD

Set 5 checked
1. A patient was admitted to the hospital with diagnosis of acute left-ventricle heart failure.
Patient’s condition suddenly became worse and edema of lungs developed in him. What kind of
disorders of peripheral blood circulation causes the lungs edema?
A. Arterial hyperemia neurotonic type
B. Arterial hyperemia neuroparalytic type
C. Arterial hyperemia metabolic type
D. Ischemia
E. Congestion (venous hyperemia)
2. Dependence of blood pressure upon level of peripheral vascular resistance was measured in
animal under experimental conditions. Indicate vessels, which have the most resistance.
A. Capillaries
B. Arteries
C. Aorta
D. Arterioles
E. Veins
3. A patient has obstruction of right calf profound veins, which results in increasing in amount of
blood in the impaired part of tissue. What is the name for increase in amount of blood resulted
from impeded blood outflow?
A. Venous hyperemia (congestion)
B. Thrombosis
C. Ischemia
D. Stasis
E. Arterial (active) hyperemia
4. A patient with paradontitis has edema of gums. His gums are dark red. What kind of local
blood flow disturbance takes place in patient’s gums?
A. Venous (passive) hyperemia
B. Arterial (active) hyperemia
C. Embolism
D. Thrombosis
E. Ischemia
5. Pain in leg at walking, cyanosis, and edema of calf appeared in a patient with varicosity. His
foot is cold. What kind of disturbances of regional blood flow appeared in this patient?
A. Angiospastic ischemia
B. Ischemic stasis
C. Compressive ischemia
D. Venous (passive) hyperemia
E. Obstructive ischemia
16
6. Edema and cyanosis of low extremities appear in a food shop assistant at the end of workday.
What is the main factor of edema development in this patient?
A. Dilatation of resistant vessels
B. Orthostatic increase of venous pressure
C. Increase of number of functional capillaries
D. Increase of collateral blood flow
E. Increase of tissue drainage
7. Redness and increase in volume of affected place of tissue, and increase in local temperature
are observed in a patient with burn of thigh. What pathologic process do indicated symptoms
correspond to?
A. Arterial hyperemia
B. Venous hyperemia
C. Thrombosis
D. Ischemia
E. Stasis
8. A 23-year-old woman had intense psycho-emotional excitement. Hyperemia of face skin,
tachycardia and increase in BP were observed in her. What is the mechanism of redness of her
face skin?
A. Congestive venous hyperemia
B. Neuroparalytic arterial hyperemia
C. Post-ischemic arterial hyperemia
D. Neurotonic arterial hyperemia
E. Stasis
9. Dyspnea, sharp pain in the chest, cyanosis, and jugular venous distention suddenly develop in
a patient with thrombophlebitis of lower extremities. What is the most possible disorder of blood
circulation developed in the patient?
A. Thromboembolism of coronary vessels
B. Thromboembolism of mesenteric vessels
C. Thromboembolism of cerebral vessels
D. Thromboembolism of portal vein
E. Thromboembolism of pulmonary artery
10. Burning in the mouth appears after taking of food with pungent dressings. At examination
mucosa of oral cavity is edematous, hyperemic, and bright red. What major reasons underlie
disorders of microcirculation?
A. Congestion
B. Neuroparalitic arterial hyperemia
C. Postischemic arterial hyperemia
D. Atonic hyperemia
E. Degranulation of tissue basophiles
11. A woman experienced a strong psycho-emotional excitement during dentist’s reception.
Redness of her face skin and profuse salivation were noticed at that time. What is the mechanism
of these phenomena?
A. Manifestation of sympathetonic effect
B. Stagnant venous hyperemia
C. Neuroparalitic arterial hyperemia
D. Postischemic arterial hyperemia
E. Neurotonic arterial hyperemia
12. Patient’s arm was put in plaster cast on account of simple fracture of humeral bone. Swelling,
cyanosis, and decrease in temperature of the traumatized arm appear next day. What disorder of
peripheral blood flow do these symptoms testify to?
A. Thrombosis
B. Venous hyperemia
17
C. Ischemia
D. Embolism
E. Arterial hyperemia
13. Injury of sympathetic fibers of the sciatic nerve developed in a patient due to trauma. What
kind of peripheral blood flow disorder takes place in the patient?
A. Neurotonic arterial hyperemia
B. Venous hyperemia
C. Angiospastic ischemia
D. Neuroparalitic arterial hyperemia
E. Obturative ischemia
14. A 42-year-old woman, shop assistant by profession, complains of edema of the lower
extremities at the end of a workday. Her legs are cyanotic, their temperature is decreased.
Venous hyperemia of lower extremities was established, which is due to constitutional weakness
of the elastic apparatus of veins and the occupation. What is the major pathogenic factor causing
local changes in venous hyperemia?
A. Disorders of metabolism
B. Hypoxia
C. Atrophy
D. Dystrophy
E. Sclerosis
15. Puncture of abdominal cavity for the extraction of fluid was performed to a 45-year-old
patient with diagnosis “cirrhosis of liver, ascites”. State of unconsciousness suddenly developed
in the patient as a result of decrease in blood pressure after extraction of 5L of fluid. That was
considered as a manifestation of brain blood circulation insufficiency. Which disorder of
microcirculation occurs in this case?
A. Ischemia
B. Arterial hyperemia
C. Venous hyperemia
D. Thrombosis
E. Embolism
16. Signs of disorders of microcirculation in form of venous hyperemia were observed in a
patient with gingivitis. This condition manifested by cyanosis and edema of mucosa and
decrease in local temperature. What is the major mechanism of congestion development?
A. Blood factors
B. Perivascular changes
C. Disturbances of lymph outflow
D. Exudation from vessels
E. Increased permeability of vessel wall
17. C. Bernard observed increase in secretion of submandibular salivary gland and development
of arterial hyperemia irritating chorda tympani (branch of nervus facialis) in an experiment.
What sort of arterial hyperemia it is according to mechanism of its development?
A. Neuroparalytic
B. Neurotonic
C. Metabolic
D. Reactive
E. Working
18. Thrombosis of coronary artery results in myocardial infarction. Which mechanisms will be
dominating ones in this disease?
A. Electrolytic-osmotic
B. Acidosis
C. Protein-dependent
D. Lipid-dependent
18
E. Calcium-dependent
19. One of the most dangerous points in myocardial infarction pathogenesis is enlargement of the
zone of necrosis, dystrophy, and ischemia. Increase in myocardial oxygen consumption plays
important role in development of indicated processes. What substances contribute to this
process?
A. Chlorine ion
B. Cholesterol
C. Catecholamines
D. Acetylcholine
E. Adenosine
20. High level of arterial blood pressure is observed in a patient with renal disease accompanied
by ischemia of renal parenchyma. What is the leading factor of increase in arterial blood pressure
in this patient?
A. Excess of angiotensin II
B. Excess of antidiuretic hormone
C. Augmentation of cardiac output
D. Increase in tonus of sympathetic nervous system
E. Hypercatecholaminemia
21. Patient has acute retrosternal pain irradiating into left arm, which cannot be controlled by the
nitroglycerine for 30 minutes. What kind of changes develop in the patient’s heart?
A. Myocardial ischemia
B. Pathological hypertrophy of myocardium
C. Sharp increase in coronary blood flow
D. Mitral valve insufficiency
E. Inflammation of pericardium
22. Severe stress was induced in an experimental animal. Under this condition, necrotic changes
of myocardium developed in the animal. What is the leading reason for pathogenesis of this
injury?
A. Increase in calcium content in cardiomyocytes
B. Decrease in mitochondrial ATP synthesis
C. Affection of Na-K pump functions
D. Coronary blood flow insufficiency
E. Increase in myosin ATP-activity
23. After fast surgical removing of coronary artery occlusion in a patient with ischemic heart
disease, secondary injury of myocardium develop (reperfusion syndrome) characterized by
necrobiotic changes in the focus of previous ischemia. This complication results from:
A. Accumulation of hydrogen ions
B. Deficiency of potassium ions
C. Deficiency of adenosine triphosphate
D. Excessive accumulation of calcium ions
E. Deficiency of creatine phosphate
24. Phlebothrombosis of the profound veins of lower extremities with subsequent pulmonary
thromboembolism develop in a patient operated for chronic ulcer of duodenum at the
postoperative period. Which functional disorder of those that develop in this case is the most
important?
A. Abrupt drop of arterial blood pressure
B. Increase in central venous pressure
C. Increase in pressure in pulmonary artery
D. Acute right ventricular failure
E. Changes of ventilation-perfusion ratio in the lungs

19
25. A 57-year-old man complains of heart pain that has developed after prolonged negative
emotions. An emergency doctor diagnosed ischemic heart disease manifesting by stenocardia.
What kind of ischemia is the most probable?
A. Compressive
B. Obliterative
C. Angiospastic resulting from deficiency of vasodilators
D. --
E. Obturative
26. Condition of a patient with thrombophlebitis of lower extremities suddenly became worse.
Weakness, giddiness, dyspnea, pain in the chest, and cyanosis developed in him. It was
established that pulmonary thromboembolism results in acute cor pulmonale. What is the leading
link in forming of this syndrome?
A. Increased blood pressure in pulmonary artery
B. Increase in central venous pressure
C. Abrupt arterial blood pressure drop in systemic circulation
D. Weakening of heart function resulting from myocardial hypoxia
E. Disorders of external respiration
27. Atophan, which leads to vessel sclerosis, was introduced to gastric arteries of an
experimental animal with purpose of gastric ulcer modeling. Which mechanism of gastric
mucosa injury is the leading one in this experiment?
A. Hypoxic
B. Neurodystrophic
C. Mechanical
D. Dysregulative
E. Nerohumoral
28. The theory exists that atherosclerosis plays an important role in periodontitis development,
affecting vessels of gums. What kind of local blood circulation disorders develops under
atherosclerosis of vessels?
A. Active hyperemia
B. Passive hyperemia
C. Embolism
D. Ischemia
E. Disorders of lymph outflow
29. Instantaneous death of pilots occurs under depressurization of airplane cabin at the altitude of
19 km. What is the reason for it?
A. Multiple gas embolisms
B. Hemorrhage to the brain
C. Gas embolism of cerebral vessels
D. Bleeding
E. Paralysis of respiratory center
30. A patient has acute pain in his chest, dyspnea, tachycardia, cyanosis, and decreased BP.
Pulmonary infarction was diagnosed in this patient. Which factor is the most common cause of
pulmonary infarction?
A. Embolism by thrombus from veins of lower extremities
B. Congestion in the pulmonary circulation
C. Increase in number of platelets
D. Activation of fibrinolytic system
E. Pneumothorax
31. A 50-year-old patient suffers from endarteritis. Which of the following symptoms is the most
characteristic one for this pathology?
A. Decreased temperature of lower extremities
B. Cyanosis
20
C. Increased temperature of lower extremities
D. Edema
E. Enlargement of volume of extremities
32. Pulmonary embolism developed in a patient with coxal bone fracture. What kind of
embolism does this patient have?
A. Fat
B. Thromboembolism
C. Tissue
D. Gas
E. Air
33. Gas embolism developed in a diver who was lifted to the surface very fast. It results from
fast change:
A. From increased atmospheric pressure to normal
B. From normal atmospheric pressure to increased
C. From normal atmospheric pressure to decreased
D. From decreased atmospheric pressure to normal
E. --
34. Acute heart arrest develops in a patient with thrombophlebitis of calf profound veins. What is
its reason?
A. Pulmonary thromboembolism
B. Left ventricle hypertrophy
C. Myocardial dystrophy
D. Endocarditis of mitral valve
E. Atherosclerosis
35. Convulsions and loss of consciousness develop in a diver during the emergency raising from
the depth. What is the major pathogenic mechanism in these disorders development?
A. Gas embolism
B. Hypoxia
C. Toxic action of oxygen
D. Toxic action of nitrogen
E. Hypercapnia
36. Destruction of endothelial layer of vessel leads to vasoconstriction. Decreased secretion of
endothelial-derived factor plays the leading role in this phenomena development. What is this
factor?
A. Nitric oxide
B. Adenosine
C. Histamine
D. Bradykinin
E. Adenosine monophosphate
37. What changes develop in human organism in case of depressurization of airplane cabin at
altitude of 19 km?
A. Gas embolism
B. Compensatory augmentation of biologic oxidation in tissues
C. Increase in body temperature
D. Compensatory augmentation of heart work
E. Disorders of muscular coordination

Set 6 checked
1. A 7-year-old child fell ill sharply. Hi has catarrh, cough, lacrimation and large-spot rash on
skin; his temperature is increased up to 38oC. Mucous membrane of patient’s pharynx is
21
edematous and red; buccal mucous membrane has whitish spots. What is the character of
inflammation underlying changes of buccal mucous membrane?
A. Hemorrhagic
B. Serous
C. Purulent
D. Catarrhal
E. Fibrinous
2. Painfulness of tooth and edema of lower part of face at the side of ill tooth are present in a
patient with acute pulpitis. What is leading mechanism of edema development in this case?
A. Increase in production of aldosterone
B. Disturbances of trophic function of nervous system
C. Disorder of neural regulation of water-salt metabolism
D. Disorders of microcirculation at the focus of injury
E. Hypoproteinemia
3. A 63-year-old woman has signs of rheumatic arthritis. Which of below enumerated indices of
blood test will be most significant for diagnosis verification?
A. R-glycosidase
B. Lipoproteins
C. Acid Phosphatase
D. Summary glycosaminoglycans
E. General cholesterol
4. A 5-year-old girl has high temperature and sore throat. At the examination of this patient the
following was revealed: edema of soft palate, gray films on tonsils, which were removed
difficulty and left after themselves deep hemorrhagic defects of tissue. Which disease of bellow
enumerated is the most possible?
A. Lacunar tonsillitis
B. Simanovsky-Vincent’s tonsillitis
C. Necrotic tonsillitis
D. Diphtheria of pharynx
E. Infectious mononucleosis
5. A cook burnt his arm with steam. What substance increased and led to development of
redness, edema and painfulness of affected area of skin?
A. Lysine
B. Histamine
C. Thiamine
D. Galactosamine
E. Glutamine
6. A child with asphyxia was taken to the hospital. On examination of the child whitish and
difficulty separating films were revealed in his larynx. What type of inflammation is observed in
this case?
A. Croupous inflammation
B. Diphtheritic inflammation
C. Purulent inflammation
D. Catarrhal inflammation
E. Serous inflammation
7. Enlargement and deformation of joints were revealed in a patient with rheumatism. What type
of inflammation underlies these changes?
A. Alterative
B. Proliferative
C. Exudative
D. Fibrinous
E. Hemorrhagic
22
8. A painful blisters full of opaque fluid has formed in patient after thermal burn. What type of
inflammation has appeared?
A. Granulomatous
B. Croup
C. Serous
D. Proliferative
E. Diphtheritic
9. Prevalence of proliferative processes was revealed in a patient with chronic inflammation of
skin and subcutaneous adipose tissue. Which hormone deficiency can lead to this situation?
A. Cortisol
B. Aldosterone
C. Insulin
D. Growth hormone
E. Thyroxin
10. A patient complains of a pain in joints. Edema and redness are visible above joints. Which
enzyme activity has to be analyzed for establishing the diagnosis?
A. Hyaluronidase
B. Creatine kinase
C. Alkaline phosphatase
D. Acid phosphatase
E. Urease
11. Indicate inflammatory mediators which have to be inhibited for decrease in exudation:
A. Catecholamines
B. Histamine
C. Heparine
D. Thromboxan
E. Interleukine-1
12. Condition of biological active substances (BAS) prevalence over there inhibitors usually
occurs in inflammation. Indicate correct correspondence of BAS to their inhibitor
A. Histamine - carboxypeptidase
B. Catecholamines - cholinesterase
C. Kinins - monoaminooxidase
D. Leukotriens - arylsulfatase
E. Serotonin-protease inhibitor
13. Edema has developed as a result of oncotic pressure elevation in the site of inflammation.
What is the main reason of hyperoncia?
A. Activation of endotheliocytes
B. Protein catabolism
C. Drainage disorders
D. Macromolecules accumulation
E. Gluconeogenesis intensification
14. What factor promotes proliferation in focus of chronic inflammation?
A. Phospholipase activation
B. Hyperosmia
C. Protein catabolism
D. Hypoxia
E. Prostacyclin synthesis
15. A patient has high body temperature, redness, edema, painfulness on her right forearm. What
biological active substances intensify inflammatory reaction?
A. Vasopressin
B. Prostacyclins
C. Phospholipase D
23
D. Proteolysis inhibitors
E. Kinins
16. A patient with inflammation of a forefinger has acute pain, edema, enlargement of local
lymph nodes, temperature rising to 38,5oC. What factors lead to exudation in the inflammatory
focus?
A. Lymph flow augmentation
B. Proliferation
C. Resistive vessels tone increase
D. Increase of volumetric flow rate of blood
E. Increasing in endothelial permeability
17. Pleurisy with sequential formation of purulent exudation developed in a patient after the
chest trauma. What is the main factor of pus formation?
A. Change of blood flow rate
B. Chemotaxis
C. Change of physicochemical properties in the site of inflammation
D. ATP production disorders
E. Hyperosmia
18. A patient has an inflammatory process on the outer surface of calf. Redness, edema, pain,
enlargement of local lymph nodes, and body temperature rising are noticed in patient. What are
the principles of pathogenetic therapy of acute inflammation?
A. Proteolysis activation blockage
B. Energy formation processes improving
C. Free radical oxidation increase
D. Introduction of mineralocorticoids
E. Introduction of thyroid hormones
19. A patient with finger inflammation has sharp pain and edema in the inflammatory focus.
After opening of an abscess these symptoms have disappeared. What favors disappearance of
pain?
A. Decrease of osmotic pressure
B. Increase in c-AMP concentration
C. Alkalosis
D. Kallikrein-kinin system activation
E. Mast cells degranulation
20. There are edema, redness and soreness in a place of injury. What is the reason for
inflammation’s symptoms appearance?
A. Sympathetic nervous system activation
B. Monoamine oxidase activation
C. BAS discharged by cells
D. Carboxypeptidase activation
E. Limited proteolysis inhibition
21. Condition of biological active substances (BAS) prevalence over there inhibitors usually
occurs in inflammation. Indicate correct correspondence of BAS to their inhibitor
A. Histamine - carboxypeptidase
B. Catecholamines - cholinesterase
C. Kinins – monoamine oxidase
D. Serotonin - arylsulfatase
E. Kallikrein - protease inhibitor
22. A 37-year-old male patient was admitted to the surgical department with symptoms of acute
pancreatitis. Which preparation is the most advisable for use in the first turn?
A. Platyphyllin
B. Contrycal
C. Analgin
24
D. Ephedrine
E. Nospanum
23. Female patient, aged 32, was stung by wasp. On the skin of left cheek (place of sting) there is
a zone of hyperemia and edema. What is the primary mechanism of edema in this case?
A. Decrease of oncotic pressure
B. Lymph drainage decease
C. Increase of oncotic pressure
D. Increase in capillary permeability
E. Increase of hydrostatic pressure in capillaries
24. What inflammatory mediator is formed due to limited proteolysis of plasma globulins?
A. Histamine
B. Leukotriens
C. Bradykinin
D. Prostaglandins
E. Lymphokines
25. Fever and increase of antibodies and leukocytes have appeared in animal under experimental
modeling of inflammation. What substances conditioned to all these common reactions in
inflammation?
A. Leukotriens
B. Interleukins
C. Mineralocorticoids
D. Glucocorticoids
E. Somatomedins
26. In a patient with eczema there are 5 typical symptoms of inflammation (Celsius -Halen
pentad). Find the one of them:
A. Cyanosis
B. Jaundice
C. Pigmentation
D. Redness
E. Albinism
27. A patient sustained trauma of knee joint and posttraumatic hemorrhagic bursitis appeared.
After 3 months passive movements limitation in extend was observed. Limitation was a result of
scar formation. What inflammatory component was a basis of this complication?
A. Secondary alteration
B. Exudation
C. Tissue hyperplasia
D. Proliferation
E. Primary alteration
28. In female patient, 28, abscess opening was performed and fast wound reparation was
observed. Reparation was preceded without score formation. What cells play the main role in
proliferation?
A. Eosinophils
B. Neutrophils
C. Fibroblasts
D. Lymphocytes
E. Monocytes
29. In examination of abscess punctate under a microscope different blood cells were revealed.
Which of them appears the first in inflammatory focus?
A. Monocytes
B. Mast cells
C. Eosinophils
D. Neutrophils
25
E. Lymphocytes
30. What is the main factor of pain formation in pulpitis?
A. BAS action
B. Hydrostatic pressure increasing
C. Excess of pain receptors excitability
D. Ischemia
E. Excess of brain centers sensitivity
31. What is the main factor of edema formation in pulpitis?
A. Capillaries permeability increasing
B. Hypoproteinemia
C. Tissue hyperosmia
D. Blood hyperosmia
E. Capillaries hydrostatic pressure increasing
32. Which of following inflammatory mediators are formatted under the influence of
lypooxygenases?
A. Leukotriens
B. Prostaglandins E1, E2
C. Prostacyclins
D. Thromboxans
E. Thrombocytes activation factor
33. Patient has a caries complicated with pulpitis. This condition is accompanied by unbearable
pain. What is the main reason of pain?
A. Ischemia
B. Emigration
C. Primary alteration
D. Exudation
E. Proliferation
34. It is known, that inflammatory mediators play the main role in inflammation pathogenesis.
What is the histamine action on inflammation?
A. Chemotaxis
B. Blood vessel permeability increasing
C. Thrombocytes aggregation
D. Blood vessels constriction
E. Blood coagulation
35. Female patient, 19, complains of pain in gums, gingival hemorrhage. These symptoms have
appeared 3 days after sulphamide drugs taking. Objectively: gingival papilli and gum margin are
reddened and swollen; there is a hemorrhage appeared in response of touching. What type of
inflammation is observed in patient?
A. Acute cattharal inflammation
B. Hemorrhagic inflammation
C. Fibrinous inflammation
D. Pus inflammation
E. Chronic cattharal inflammation
36. A patient 6 years old was admitted to the hospital with asphyxia (oxygen deficiency). The
membranous coats on mucosa of fauces were revealed. The coats can be removed easy. What
type of inflammation is in patient?
A. Fibrinous
B. Necrotic
C. Pus
D. Cattharal
E. Hemorrhagic

26
37. In child patient there are following symptoms of inflammation over the skin scratch: pain,
swallowing, redness. These sings are symptoms of immediate hypersensitivity. What blood cells
lead to such changings?
A. Lymphocytes
B. Eosinophils
C. Basophiles
D. Monocytes
E. Neutrophils
38. In a girl patient, 5, there are intense throat pain and fever. Objectively were revealed edema
and membranous coats on mucosa of fauces. The coats cannot be removed easy. What of the
followings are more probably?
A. Simanovsky - Vensann tonsillitis
B. Infectious mononucleosis
C. Diphtheria of fauces
D. Necrotic tonsillitis
E. Lacunar tonsillitis
39. Patient has a caries complicated with pulpitis. This condition is accompanied by unbearable
pain. What is the main factor of pain appearing?
A. High pressure of fluid in inflammatory area
B. Bradykinin
C. Prostaglandins
D. Histamine
E. Serotonin
40. There are lots of neutrophils in abdominal pus exudates in a patient with peritonitis. What is
the main function of neutrophils in inflammatory area?
A. Phagocytosis
B. Prostaglandins secretion
C. Degranulation
D. Histamine liberation
E. Local blood flow regulation
41. Keloid cicatrix has formatted in a place of abscess. Name the stage of inflammation, which
caused scar appearing.
A. Proliferation
B. Exudation
C. Primary alteration
D. Secondary alteration
E. Emigration
42. A male patient, 16, was admitted to the hospital with acute appendicitis. What typical
pathological process is the basis of this disease?
A. Inflammation
B. Hypoxia
C. Fever
D. Tumor
E. Allergy
43. Eyeball inflammation resulted pus in the anterior chamber of the eye (hypopyon). What stage
of inflammation was it?
A. Exudation
B. Proliferation
C. Primary alteration
D. Secondary alteration
E. Emigration

27
44. In a patient with skin pathological process which has 5 typical symptoms: tumor, rubor,
calor, dolor, functio laesa. The development of such process includes 3 stages:
Alteration, .......... , proliferation. What is the second one?
A. Exudation
B. Regeneration
C. Reparation
D. Fibrosis E. Petrification
45. The secondary alteration is caused by:
A. Lysosomal enzymes
B. Kinins
C. Lympokynes
D. Prostaglandins
E. Complement components C3a and C5a
46. In proliferation cells multiplying and defect compensation take place. What is the reason of
cells growth in specific clones?
A. Keylones
B. Platelet-derived grow factor
C. Interleukins
D. Somatomedine
E. Lymphocytes peptide
47. There are following components in pleural effusion: proteis-62g/l, abundance of neutrophils.
What is the type of inflammation in this case?
A. Serous
B. Suppuratuve
C. Fibrinous
D. Hemorrhagic
E. Putrefactive
48. Necrotic focus has appeared on burn, swallowing and red skin. What is the main mechanism
of necrobyosis improvement in inflammatory area?
A. Secondary alteration
B. Primary alteration
C. Emigration of leucocytes
D. Dyapedesis of erythrocytes
E. Fibroblasts proliferation
49. Glucocorticoid treatment is usefull in rheumatic arthritis. What is the basal action of
glucocoricoids?
A. Histamine decreasing
B. Emigration inhibition
C. Phagocytosis inhibition
D. Prostaglandins synthesis inhibition
E. Cell membranes stabilization
50. A patient with abscess of finger has some local and general symptoms of inflammation. What
from following is a local inflammatory sing?
A. Swelling
B. Fiver
C. Headache
D. Lymphangitis
E. Leucocytosis
51. It is known that inflammatory mediators are cellular or humoral. What from the following is
a humoral mediator?
A. Kallidine
B. Histamine
28
C. Serotonins
D. Interleukine-2
E. Hydrogenium peroxide

Set 7 checked
1. A 52-year-old woman complains to bad pain around her abdomen and back. Acute
appendicitis was diagnosed. There is suspicion of complication of pancreatitis by abscess. Which
type of fever would corroborate appearance of abscess?
A. Hectica
B. Continuous
C. Remittent
D. Intermittent
E. Atypical
2. Polyuria developed in a patient with fever after period of olyguria. What is the leading factor
of diuresis change at the end of the second period of fever?
A. Normalization of hormonogenesis by adrenal medulla
B. Normalization of hormonogenesis by reticular zone of adrenal cortex
C. Normalization of hormonogenesis by glomerular zone of adrenal cortex
D. Normalization of hormonogenesis by fascicular zone of adrenal cortex
E. Normalization of thyroid gland function
3. A patient has felt cold, chills, “goose flesh”, increase of body temperature. Which else
changes characterize the first period of rapid elevation of body temperature?
A. Tachycardia
B. Equilibration between heat production and heat loss
C. Dilation of skin vessels
D. Decrease of arterial pressure
E. Increase of metabolism on 100-200%
4. Body temperature of patient becomes pyretic. Which substances have to act to neurons of
thermoregulation for fever development?
A. Interferon
B. Kallidinum
C. Prostaglandins
D. Free radicals
E. Leucotriens
5. Fever in a patient develops in following succession of stages:
A. Incrementi; fastigii; decrementi
B. Incrementy; decrementy; fastigii
C. Fastigii; decrementi; incrementi
D. Fastigii; incrementi; decrementi
E. Decrementi; fastigii; incrementi
6. The temperature of a patient with infectious disease increased to 39.5-40.5 0C in a day and
kept that level about 1 hour, but then it returned to the normal level. Which type of fever is
described in that case?
A. Continuous
B. Intermittent
C. Remittent
D. Recurrent
E. Atypical

29
7. The body temperature of a patient with infectious disease increased to 39.5-40.5 0C in a day
and kept this level about 1 hour, but then became normal again. Which disease is characterized
by described type of fever?
A. Tuberculosis
B. Influenza
C. Peritonitis
D. Brucellosis
E. Malaria
8. A patient had fever after injection of pyrogenal. His skin has become pale, cold; chill appeared
in him, oxygen consumption increased. How do the processes of thermoregulation change in
described period of fever?
A. Increase of heat production and decrease of heat loss
B. Decrease of heat loss
C. Heat loss is equal heat production
D. Decrease of heat production and increase of heat loss
E. Decrease of heat production
9. Body temperature of the 8-year-old Sasha with meningitis was on the level 39-40 0C for a
week. After 8 days under the influence of antibiotics the temperature has decreased to 36.8 0C in
1.5 hours. There were acute hyperemia of skin, profuse sweating, decrease of arterial pressure,
and loss of consciousness in him. Which medicine is the pathogenetic remedy?
A. Vasoconstrictors
B. Antibiotics
C. Antipyretics
D. Pyrogenal
E. Sulfanilamide
10. After being in the room with air temperature 40 0C and humidity 80% a patient has been
brought to hospital in grave condition. He was unconscious; he had tachypnea, tachycardia, and
body temperature 41 0C. Reanimation was failed. The patient has died. What is the most possible
direct reason of death in this case?
A. Paralysis of the breath center
B. Collapse
C. Coagulation of blood and decrease of volume of circulating blood
D. Dehydration
sE. Heart failure
11. Fever of the patients with relapsing fever is characterized by several periods of pyretic
temperature per day and several periods of normal temperature. Such type of temperature curve
is called:
A. Febris hectica
B. Febris intermittens
C. Febris continua
D. Febris recurrens
E. Febris atypica
12. Sharp increase of the temperature to 38.7 0C was marked in a patient with acute purulent
periodontitis. His body temperature has decreased to normal level after opening the pulp cavity.
Which type of fever was in the patient?
A. Efemeral
B. Septic
C. Recurrent
D. Remittent
E. Continua

30
13. A patient suffers from osteomyelitis of maxilla. His body temperature increases to 40 0C and
then sharply decreases to 35.6 0C every day. Which type of fever curve is characterized by these
changes?
A. Continua
B. Intermittent
C. Reccurens
D. Atypica
E. Hectica
14. Pallor of the skin, “goose flesh” and increase of oxygen consumption appeared in the
patient’s skin after injection of pyrogenal. Which stage of fever is characterized by these
changes?
A. Stadium incrementi
B. Stadium fastigii
C. The stage of falling temperature by crisis
D. The stage of falling temperature by lysis
15. Acute increase of body temperature, dyspnea, tachycardia, nausea, convulsions, and loss of
conciousness developed in a worker, working in the thick uniform in summer. What was the
most possible reason of development of those symptoms?
A. Equilibration between heat loss and heat production
B. Decrease of heat production
C. Decrease of heat loss
D. Increase of heat production
E. Increase of heat loss
16. A man in light clothes is staying in a room with air temperature +14 0C. Windows and doors
are closed. Which way of heat loss is the most considerable in this case?
A. Evaporation
B. Perspiration
C. Conduction
D. Radiation
E. Convection
17. Experimental mice were kept in a lodge with air temperature 4 0C. Which adaptive reaction
supplies its thermal homeostasis?
A. Limitation of heat loss
B. Decrease of oxygen consumption
C. Anabiosis
D. Increase of blood consumption
E. Decrease of oxidation enzyme activity
18. Inclination of the set point of thermoregulation to higher level due to action of IL-1 is in a
patient. What is the name of this typical pathological process?
A. Fever
B. Hyperthermia
C. Hypothermia
D. Inflammation
E. Hypoxia
19. The body temperature of a patient with crupous pneumonia is 39 0C. The difference between
the morning and evening temperature of his body didn’t exceed 1 0C during 9 days. Which type
of the fever curves was that?
A. Continua
B. Hectica
C. Intermittent
D. Hyperpyretic
E. Reccurens
31
20. A patient has fever with following stages: incrementi, fastigii, decrementi. Which disease
these features can characterize?
A. Acute pneumonia
B. Acromegaly
C. Diabetes mellitus
D. Hyperaldosteronism
E. Myocardial hypertrophy
21. Body temperature of patient is pyretic; his skin is hot and red. What was the correlation
between processes of heat production and heat loss in described stage of fever?
A. Heat loss is equal heat production
B. Heat loss is more then heat production
C. Heat production is more then heat loss
D. Heat loss is less then heat production
E. Heat production is less then heat loss
22. The body temperature of a patient with pneumonia was keeping on the level 38.3-38.5 0C all
the first week of disease. Such fever is called:
A. Febrile
B. Hyperpyretic
C. Pyretic
D. Subfebrile
23. Increase of “acute phase” proteins level in blood such as ceruloplasmin, fibrinogen, C-
reactive protein is typical for development of fever. Indicate the possible mechanism of this
phenomenon.
A. Stimulative influence of IL-1 on hepatocytes
B. Destructive action of elevated temperature to the cells of the organism
C. Proliferate action of IL-2 to T-lymphocytes
D. Adaptive reaction of the organism to pyrogen
E. Degranulation of mast cells
24. Most infectious diseases are characterized by development of fever. It can be explained:
A. Formation of IL-1 during phagocytosis of microorganisms
B. Intoxication of the organism
C. Degranulation of mast cells
D. Activation of T- and B-lymphocytes
E. Processes of exudation
25. Attacks of fever in a patient occur periodically. During the attack the body temperature
sharply increases, keeps pyretic level nearly 2 hours and then decreases to normal level. This
type of fever is typical for:
A. Tertian fever
B. Sepsis
C. Brucellosis
D. Relapsing fever
E. Epidemic typhus
26. Body temperature of a patient is 39 0C for several hours (stadium fastigii). Indicate which
changes of physiological functions are the most typical for this stage of fever.
A. Bradycardia
B. Ingibition of phagocytosis
C. Increase of heat production
D. Increase of heat production
E. Heat production is equal heat loss
27. In a patient with pneumonia the increased body temperature was revealed. What sort of
biological active substances play the most important role in development of fever?
A. Interleukin-1
32
B. Leukotriens
C. Serotonin
D. Histamine
E. Bradykinin
28. In a patient with prolonged fever after the course of treatment body temperature begins
decreasing. What is the possible mechanism of temperature decrease?
A. Protective activation of immune system
B. Decrease of heat production due to reducing metabolic rate
C. Decrease of production of pyrogens
D. Increase of resistance of organism to action of the pyrogens
E. Increase of heat loss due to peripheral vasodilatation
29. What is the most efficient mechanism of heat loss in case of following environmental
conditions: 80% air humidity and +35 0C (95 degree Fahrenheit) air temperature?
A. Hyperventilation
B. Sweating
C. Conduction
D. Convection
E. Irradiation
30. During the examination of the patient following symptoms were revealed: redness of skin,
skin is hot and dry to touch, heart bit rate is 92 per minute, respiratory rate is 22 per minute, body
temperature is 39,2 0C (102.5 degree Fahrenheit). What is the correlation between heat
production and heat loss in described period of fever?
A. Heat production surpasses heat loss
B. Heat production equals to heat loss
C. Heat production is less than heat loss
D. Decreasing of heat production without changes of heat loss
E. Increasing of heat production without changes of heat loss
31. Animal was injected with pyrogen to reproduce fever. What mechanism starts process of
temperature increasing?
A. Activation of non-shivering thermogenesis
B. Activation of shivering thermogenesis
C. Rise of set point of thermoregulation in hypothalamus
D. Reduction of heat loss
E. Dissociation of oxidation and oxidative phosphrilation in tissues
32. In a patient with third stage of fever reaction following manifestations observed: abundant
sweating, tachypnea (increase in respiratory rate), decrease of body temperature. What is the
mechanism of development of these symptoms?
A. Reduction of shivering thermogenesis
B. Secondary aldosteronism
C. Rise of set point of thermoregulation in hypothalamus
D. Predomination of heat production over the heat loss
E. Peripheral vasodilatation
33. Patient, who has been suffering from malaria, has weakness of heart activity and tachycardia
during the stage of heart loss. What is the name of this complication?
A. Infectious-toxic collapse
B. Bacterial shock
C. Hemorrhage collapse
D. Ortostatic shock
E. Hemorrhage shock
34. Adaptation of organism disturbances to decrease of environmental temperature when using
medicines alpha-adrenoblokators. Which mechanism is responsible for this?
A. Formation of primary heat
33
B. Constriction of skin vessels
C. Contractile thermogenesis
D. Sweating
35. After blood trasfusion patient complaints feeling of heat, rigor, increase of body temperature
to +40 0C. Its known the cause of elevation temperature is secretion of endogenous pyrogens.
Which cells produce endopyrogens?
A. Erythrocytes
B. Platelets
C. Endotheliocytes
D. B-lymphocytes
E. Macrophages
36. The different types of temperature curve are reported on examination of the infected patients.
What other pathological conditions can lead to the fever?
A. Isoosmotic hyperhydratation
B. Sistemic immune complex diseases
C. Excess production of glucocorticoids
D. Protein starvation
E. Hypohonadism

Set 8 checked
1. Pain in the heart and joints and pneumonia appeared in a patient three weeks later acute
myocardial infarction. What is the main mechanism of development of postinfarction Dressler’s
syndrome?
A. Ischemia of myocardium
B. Resorption of enzymes from necrotized area of myocardium
C. Secondary infection
D. Thrombosis of vessels
E. Autoimmune inflammation
2. A patient addressed to a dentist with complaints of redness and edema of mucous membrane
of his mouth a month later dental prosthesis. Allergic stomatitis was diagnosed in this patient.
What type of allergic reaction by Gell and Cumbs underlies this disease?
A. Cytotoxic
B. Delayed-type hypersensitivity
C. Immune complex-mediated
D. Anaphylactic
E. Stimulating
3. Anaphylactic shock developed in a patient with botulism after second injection of antitoxic
antibotulinus serum mixture. What is the main mechanism of anaphylaxis?
A. Interaction of T-lymphocytes with mediators
B. Interaction of antigen with IgM
C. Interaction of macrophages with antigens
D. Interaction of antigen with IgE
E. Interaction of T-lymphocytes with tissue basophils
4. In a 27-years-old man tuberculin test was carried out. Following was observed 24 hours later:
infiltration with size of 40x35 mm at the site of injection and hyperemia of skin above it. What
group of biologic active substances causes development of allergic inflammation in this patient?
A. Lymphokines
B. Biogenic amines
C. Prostaglandins
34
D. Leukotriens
E. Kinins
5. Purulent endometritis developed in a woman after delivery. Treating with antibiotics -
inhibitors of murein synthesis was ineffective. Wide spectrum bactericidal antibiotic was
administered to her. 6 hours later temperature rapidly increased up to 40oC with shivering, pains
in muscles appeared, BP dropped down to 70/40 mmHg, and oligura developed in this woman.
What is the main reason for this condition development?
A. Endotoxic shock
B. Toxic effect of preparation
C. Internal bleeding
D. Anaphylactic shock
E. Bacteremia
6. A 24-years-old patient has edema of face and increase in BP, which appeared 1.5 weeks later
severe streptococcus tonsillitis. The patient has hematuria and proteinuria of 1.2 g/L. Anti-
streptococcus antibodies and decrease in content of compliment system components were
revealed in patient’s blood. Which microvessels do deposits of immune complexes localize in
and cause nephropathy?
A. Proximal tubules
B. Glomerule
C. Descendent tubules
D. Loop of Henle
E. Pyramids
7. Nausea, fatigue, stomachache, palpitation, difficult respiration, and skin blisters developed in
a patient 25 minutes later injection of antibiotics. What stage of allergic reaction is observed in
this patient?
A. Pathochemical
B. Biochemical
C. Pathophysiological
D. Immunological
E. Sensibilization
8. Skin tuberculin test was carried out in a patient with chronic lung tuberculosis. Local
hyperemia and edema appeared in the site of inracutaneous introduction of tuberculin
preparation within 24-48 hours. What cells are primary effectors in mechanism of this reaction?
A. Neutrophils
B. T-lymphocytes
C. B-lymphocytes
D. Endotheliocytes
E. Smooth muscle cells of microvessels
9. Hyperemia, swelling and then necrosis of tissue, their rejection and ulcer (Arthus
phenomenon) develop at the rabbit in the place of secondary intracutaneus injection of a
substance with strongly pronounced antigenic properties (for example horse serum). What
factors play the main role in pathogenesis of this phenomenon?
A. Antibodies presented by IgE
B. Antibodies presented by IgD
C. Antibodies presented by IgA
D. Antibodies presented by IgG and/or IgM
E. Specific T-lymphocytes-effectors
10. Skin rash, itching, swelling and pain in joints, increase in body temperature, and proteinuria
appeared in a patient in 5-8 days after use lots of medical serum. Serum sickness was diagnosed.
What is the main factor in pathogenesis of this syndrome?
A. Primary systemic accumulation of circulating immune complexes in the blood
B. Primary systemic degranulation of mast cells in the organism
35
C. Primary systemic activation of T-killers
D. Primary systemic activation of endoteliocytes
E. Primary systemic cytolysis of blood cells
11. Allergic diagnostic tests are used for the diagnosis of many infectious diseases (tuberculosis,
brucelosis, tularemia etc). Diagnosis is confirmed if papula and redness appear in the place of the
allergen injection. Antigens interaction reaction is conditioned by:
A. IgE and lymphokines
B. IgM and macrophages
C. T-lymphocytes and lymphokines
D. IgE and T-lymphocytes
E. IgM and tissue basophiles
12. Catarrhal inflammation of bulbar conjunctiva and nose mucous membrane develop in patient
every year in spring and early summer, when trees and flowers are in blossom. Production of
specific antibodies to pollen underlies this syndrome. What cells activate and develop exocytosis
in this syndrome?
A. Neutrophils
B. Macrophages
C. Lymphocytes
D. Mast cells
E. Throbocytes
13. An 18-year-old man with shoulder phlegmon got intramuscular injection of penicillin.
Tachycardia, thread-like pulse; decrease in BP down to 80/60 mmHg occur after that. What kind
of pharmacologic reaction develops?
A. Potentiation
B. Reflex action
C. Central action
D. Anaphylaxis
E. Peripheral action
14. Patient has been wearing removable dental prosthesis for 7 days. Soreness, swelling and
hyperemia of prosthesis bed appear in the patient after that. Inflammation subsides after
prosthesis taking off and develops again after prosthesis putting on. What process underlies the
inflammation development?
A. Mechanical irritation
B. Hypersalivation
C. Hyposalivation
D. Resorption
E. Secretion
15. Thyrotoxicosis was diagnosed in a patient. Antithyroid antibodies were found in his blood.
Which type of allergic reaction is observed at development of this disease?
A. Immune complex-mediated
B. Stimulating
C. Anaphylactic
D. Cytotoxic
E. Delayed type hypersensitivity
16. Hives, itching of the skin, swelling of the skin and mucous membranes, swelling of
lymphatic nodes develop in the patient in 9 days after injection of medicinal serum. What disease
develops?
A. Pollinosis
B. Serum sickness
C. Shwartzman’s phenomenon
D. Overy phenomenon
E. Quincke’s edema
36
17. Dressler’s syndrome was diagnosed at the patient 1.5 month later myocardium infarction. It
is characterized by pericarditis, pleurisy, and pneumonia. What is the reason for this syndrome?
A. Sensitization of the organism by myocardium antigens
B. Decrease in resistance to microorganisms
C. Activation of saprophytic microflora
D. Intoxication of organism by products of necrosis
E. Release of myocardial enzymes to the blood
18. It is known that bronchial asthma develops by mechanism of immediate hypersensitivity,
which includes 3 sequential stages:
A. Immunological, pathochemical, pathophisiological
B. Pathochemical, pathophisiological, immunological
C. Pathochemical, immunological, pathophisiological
D. Pathophisiological, immunological, pathochemical
E. Pathophisiological, pathochemical, immunological
19. A 20-year-old man has injury of the right testicle. What danger does it brings for the left
(healthy) testicle?
A. Mimicry of antigens and development of antibody-mediated damage
B. Development of infectious process
C. Development of atrophy
D. Development of hypertrophy
E. No danger
20. Guinea-pig’s nephrocytotoxic serum was injected to the rabbit under the experiment. What
human disease is modeled in this case?
A. Nephrotic syndrome
B. Acute pyelonephritis
C. Chronic renal insufficiency
D. Acute diffuse glomerulonephritis
E. Chronic pyelonephritis
21. Acute glomerulonephritis appeared in the patient 2 weeks later purulent tonsillitis.
Antibodies against microorganism antigens were found at the patient. Which microorganism are
these antibodies against?
A. Hemolytic streptococcus
B. Staphylococcus
C. Pneumococcus
D. Mycobacterium tuberculosis
E. Meningococcus
22. Severe edema of soft tissues of upper and lower jaws, rash on the skin of face, redness, and
itching appear in the patient in response to using anesthetic drag at tooth extraction. Which
pathological process underlies the reaction to anesthetic?
A. Inflammation
B. Drag toxic action
C. Allergy
D. Insufficiency of blood circulation
E. Disorder of lymph outflow
23. Novocain was injected by dentist for anesthesia at tooth extraction. Symptoms of
anaphylactic shock appeared at the patient few minutes later. Patient has drop of BP, tachypnea,
loss of consciousness and convulsions. What type of reaction is it?
A. Immediate type hypersensitivity
B. Cytolytic or cytotoxic reactions
C. Arthus phenomenon reactions
D. Delayed type hypersensitivity
E. Stimulating allergic reaction
37
24. Man with the caries is subjected to constant sensitization by streptococcus antigen. What
disease can appear due to this etiological factor?
A. Glomerulonephritis
B. Pancreatitis
C. Myocarditis
D. Pulpits
E. Periodontitis
25. Antitoxic diphtheria serum was introduced to a child suffering from diphtheria. Skin eruption
accompanied by itching, increase in body temperature to 38oC, and pain in joints occurred in
patient 10 days later. What is the reason for these symptoms?
A. Contact allergy
B. Serum sickness
C. Atopy
D. Anaphylactic reaction
E. Delayed type hypersensitivity
26. Tuberculin was injected intraperitoneally to animal, which was sensitized by it. Venous
hyperemia and peritoneal edema were revealed 24 hours later in laparotomy. High quantity of
lymphocytes and monocytes were found in peritoneum smears. What pathological process does
the animal have?
A. Fibrinous inflammation
B. Allergic inflammation
C. Aseptic inflammation
D. Serous inflammation
E. Purulent inflammation
27. A dentist injected ultracain to a patient before tooth extraction for the purpose of anesthesia.
Sensitivity test was not made. Anaphylactic shook developed in the patient in few minutes after
drag injection. What cells produce reagins, which take part in development of anaphylactic
reaction?
A. Plasma cells
B. B-lymphocytes
C. T-lymphocytes
D. Mast cells
E. Eozinophiles
28. Tooth was extracted in a teenager under Novocain anesthesia. Paleness of skin, dyspnea and
hypotension occurred in the patient 10 minutes later. What type of allergic reaction is it?
A. Anaphylactic
B. Cytotoxic
C. Arthus phenomenon type
D. Delayed type hypersensitivity
E. Stimulating
29. Tooth was extracted in a teenager under Novocain anesthesia. Paleness of skin, dyspnea and
hypotension occurred in the patient 10 minutes later. What substance does allergen react with on
the surface of mast cells?
A. IgE
B. T-lymphocytes
C. IgA
D. IgD
E. IgM
30. A 27-year-old woman instilled drops with penicillin to the eyes. Itching and burning pain of
skin, edema of lips and eyelids, cough with whistling, decreasing in BP occur in her some
minutes later. What immunoglobulin participates in development of these allergic reactions?
A. IgE
38
B. IgG and IgM
C. IgA
D. IgM
E. IgG
31. A 43-year-old woman is suffering from pneumonia. She began complaining of weakness,
face and arms burning pain in 10 minutes after ampicillinum injection. Cough, dyspnea, pain in
the chest developed in her. At clinical examination patient has cyanosis, eyelids swelling, face
red rashes, heart rate – 120 per minute, BP – 120 mmHg, muffled heart sounds, hypopnoe and
tachypnea, respiration with different moist rales. What is the reason for worsening of patient’s
condition?
A. Anaphylactic shook
B. Urticaria
C. Quincke’s edema
D. Attack of asthma
E. Pulmonary thromboembolism
32. 0.1 ml of horse serum was injected to the guinea-pig for sensitization. What are external
signs of sensitization?
A. No external signs
B. Skin rashes
C. Joints swelling
D. Increase in body temperature
E. Pain
33. A 15-year-old girl is suffering from bronchial asthma. Severe attack of expiratory dyspnea
develops in her during the spring blossoming period. What biological active substance causes
spasm of bronchial smooth muscle in this case?
A. Leukotriene
B. Thromboxane A2
C. Prostacyclin
D. Bradykinin
E. Serotonin
34. Patient addressed to a doctor with complaints of headache, rhinitis, weakness, and increase in
body temperature developing every spring in blossoming period. What type of allergic reaction
by Gell and Cumbs underlies this disease?
A. Anaphylactic
B. Cytotoxic
C. Antibody-dependent cell-mediated cytotoxicity
D. Immune complex-mediated
E. Delayed type hypersensitivity
35. Weakness, itching of skin, acute spasmodic pain in the abdomen, hyperemia and rash on
skin, tachycardia, and decrease in BP down to 70/40 mm Hg suddenly developed in a patient few
minutes later Novocain injection by the dentist. Which type of allergic reactions does this
pathology belong to?
A. Anaphylactic
B. Cytotoxic
C. Stimulating
D. Cell-mediated
E. Immune complex-mediated
36. Contact dermatitis of upper extremities developed in the nurse, who has been working in
manipulation room for 20 years. Which type of allergic reactions does this pathology belong to?
A. Delayed-type hypersensitivity
B. Primary immunodeficiency
C. Immediate type hypersensitivity
39
D. B-cells immunodeficiency
E. T-cells immunodeficiency
37. Pain in the joints and loins, hemorrhagic eruptions on the skin, and increase in body
temperature occur in the patient with toxemic stage of burn disease in 2 hours after allogenic
plasma transfusion. What allergic reaction takes place in this case?
A. Serum sickness
B. Urticaria
C. Quincke’s edema
D. Anaphylactic
E. Autoimmune vasculitis
38. Hyperergic inflammation form of upper respiratory tract (larynx, trachea, bronchi) develops
at 6-year-old child. Threat of respiratory impairment develops and then necessity of using anti-
inflammatory hormones occurs. Which hormone has anti-inflammatory property?
A. Cortisol
B. Adrenaline
C. Growth hormone
D. Testosterone
E. Insulin
39. Swelling, increased BP, proteinuria, hematuria, decreased urine excretion were revealed in
the patient, which suffered severe tonsillitis before. These symptoms are typical of acute
glomerulonephritis, resulting from damage of glomerular basement membrane. What is
mechanism of this disease?
A. Anaphylactic allergic reactions
B. Cytotoxic allergic reactions
C. Immune complex-mediated allergic reaction
D. Delayed-type hypersensitivity
E. Stimulating allergic reaction
40. An 18-year-old patient has insulin- dependent diabetes mellitus I type. What type of allergic
reactions underlies beta cells damage?
A. II type, cytotoxic
B. I type, anaphylactic
C. III type, immune complex-mediated
D. IV type, delayed-type hypersensitivity
E. Pseudoallergic reaction

Set 9 checked
1. Rate of appearance of tumors is increased in elderly people. One of the main reasons for this
is:
A. Increase in activity of cellular immunity
B. Decrease in activity of cellular immunity
C. Rise of disorders of mitoses
D. Decrease of intensity of antibody production
E. Increase of intensity of antibody production
2. In a patient with metastases of lung carcinoma introduction of cytostatics led to suspension of
metastases growth at first but later metastases resumed spread. What is the most possible
mechanism of secondary growth of metastases?
A. Absence of contact braking
B. Absence of Heiflik’s limit
C. Rise of genetic heterogeneity of tumor cells
D. Increased glucose consumption by tumour

40
E. Increased amino acids consumption by tumour
3. A 56-years-old patient, who had contact with diethylnitrozamine at his work place, complains
of pain in right subcostal area, weakness, loss of appetite, and decreased workability. At
examination of this patient: surface of his liver is rough, splenomegaly and ascites are present in
him; his body temperature is 37.2oC; in his blood analysis ESR is 25 mm/hour, besides
neutrophilic leukocytosis, and hypochromic anemia were found. What disease developed in the
patient’s organism?
A. Cancer of liver
B. Hepatitis
C. Cirrhosis of liver
D. Gallstone disease
E. Dyskinesia of bile ducts
4. Approximately 60% of tryptophan is oxidized through serotonin pathway in case of malignant
intestine carcinoma. What vitamin demand is increased for a patient with malignant carcinoma
of intestine?
A. Nicotinic acid
B. Pantothenic acid
C. Folic acid
D. Pyridoxine
E. Riboflavin
5. Malignant tumor of lung was diagnosed in a patient. What feature of tumour growth testifies
its malignancy?
A. Infiltrative growth
B. Unregulated growth
C. Unlimited growth
D. Expansive growth
E. Appearance from one cell
6. Tumor has developed in a patient with innate immunodeficiency. What factor of non-specific
immunity participates in anti-tumor defense?
A. Lysozyme
B. Interleukin-1
C. Lactoferrine
D. Arylsulfatase
E. Properdine
7. The woman complained to the doctor for changing of voice, appearance of hair on the face,
and reduction of breast. Where would a tumor develop that could lead to these symptoms?
A. Tumor of zona reticulata of adrenal glands
B. Tumor of ovaries
C. Tumor of anterior lobe of pituitary gland
D. Tumor of zona glomerulata of adrenal glands
E. Tumor of zona fasciculata of adrenal glands
8. Gamma-interferon was used for treatment of the patient, suffering from tumor. What property
of this substance was used for treatment of this tumor disease?
A. Activation of synthesis of Ig
B. Activation of B-lymphocytes
C. Activation of killer effect
D. Activation of complement system
E. Activation of macrophages
9. Following changes can occur in development of tumor:
A. Pheochromocitoma – hypotension
B. Insulinoma – hypoglycemia
C. Aldosteroma – hypohydratation
41
D. Tumor of zona reticulata of adrenal glands – inhibition of sexual growth
E. Tumor of thyroid gland – hypothyroidism
10. Cancer of lung developed in a patient, smoking for a long time. Which cancerogenous
substance is present in tobacco smoke, related to polycyclic aromatic carbohydrates?
A. Benspyren
B. Dimethylaminobenzol
C. Beta-naphthylamine
D. Dietilnitrozamine
E. Orthoaminoazotoluol
11. A male patient, 40, has stenotic (without metastases) esophagus cancer. The following
changes were revealed in that patient: muscular and fat tissue atrophy, brownish color of the
skin, thin epidermis, and cardiac atrophy. What’s the reason of such symptoms?
A. Alimentary cachexy
B. Myasthenia
C. Addison’s disease
D. Cancer cachexy
E. Brown induration
12. In 1910 Rhauss managed to cause sarcoma in chickens by cell-free infiltrate inserting. What
was the method of experimental modeling?
A. Induction
B. Explantation
D. Homotransplantatio
C. Isotransplantation
n
E. Heterotransplantation
13. There is high stage of interaction between lung cancer and tobacco smoking. What chemical
carcinogen is contained in tobacco smog?
A. 3,4-benspyren
B. Orhtoaninotoluol
C. Aphlatoxin
D. Methylcholatren
E. Dyethylnitrosamine
14. Patient complained of weight loss and weakness, in blood analysis hypoglycemia and
hyperinsulinemia were revealed. An additional study tumor of beta-cells was discovered. Insulin
synthesis improvement in this case is a result of:
A. Biochemical athypia
B. Morphological athypia
C. Functional athypia
D. Differentiation athypia
E. Immunological athypia
15. What biological process augmentation is typical for tumor cells?
A. Anaerobic glycolysis
B. Decarboxilation
C. Tissue respiration
D. Lipolysis
E. Gluconeogenesis
16. Erlich’s tumor was transplanted to animal. What is the evidence of tumor progression?
A. Unlimited growth
B. Resistance to cytostatics
C. Anaplasia
D. Infiltration
E. Tumor weight increasing
42
17. Unpainted formation under the jaw was appeared in liquidator of Chernobyl’s disaster after
12 years of accident. The size of it has increased till last month. The blood analysis is in norm.
What pathological process is most suspicious in this case?
A. Lymphadenitis
B. Syaloadenitis
C. Abscess
D. Malignant tumor
E. Cyst
18. What is the most effective manner of experimental transplantation of tumor?
A. Isotransplantation
B. Autotransplantation
C. Heterotransplantation
D. Homotransplantation
E. Allotransplantation
19. There is a tumor of tongue in patient. What characteristics of tumor may be considered as
malignant?
A. Dysplasia
B. Expansion
C. Infiltration
D. Pasteur’s positive effect
E. Mitosis increasing
20. What cell structure is a «target» for chemical cancerogens?
A. Nuclear DNA
B. Lysosomes
C. Mitochondria
D. Cytoplasmic membrane
E. Ribosomes
21. A patient with lung cancer has been smoking 30 cigarettes per day for 20 years. What the
group of cancerogens is in tobacco smog?
A. Polycyclic carbohydrates
B. Aminoasosubstances
C. Nitrosamines
D. Amines
E. Heterocyclic carbohydrates
22. A female patient was admitted to the hospital with diagnosis «uterine cervix cancer». What
from the following processes are not typical for tumor cells?
A. Anaerobic glycolysis
B. High glucose metabolism
C. Aerobic glucolysis
D. High aminoacides metabolism
E. Organospecificity absence in aminoacides consumption
23. A patient with urinary bladder cancer was working in coke factory. What substance was the
most probable reason of this pathological condition?
A. Naphtylamine
B. Dichlorethane
C. Vinegar acid
D. Alcohol
E. Pethroleynic aether
24. It is established that tumor tissue receives in 20-25 times less of glucose that intact tissue in
equal glucose amount. What metabolic changings lead to such event?
A. Aerobic glycolysis enhancement
B. Oxydation improvement
43
C. Normal interaction of these processes
D. Tissue respiration improvement
E. Decreasing of anaerobic glycolysis
25. They got nitrogenous nitrite to experimental animals. A tumor was developed in 80% of
animals. What was the group of cancerogens?
A. Nitrosamines
B. Aminoasosubstances
C. Polycyclic carbohydrates
D. Simple chemical substances
E. Hormones
26. After Chernobyl disaster morbidity of tumors has been increasing. What action of the
radiation has been appearing?
A. Oncogenic
B. Thermal
C. Mutagenic
D. Cytostatics
E. Immunostimulative

Set 10
1. A patient was admitted to the hospital in comatose state. Accompanying people said that hi
lost consciousness at training while he was finishing the Marathon distance. What sort of coma is
the most possible in this patient?
A. Hyperglycemic
B. Hypoglycemic
C. Hypothyroid
D. Hepatic
E. Diabetic ketoacidosis
2. Patient has hyperglycemia, glucosuria, polydipsia, polyphagia, and polyuria. What hormone
hyposecretion do these changes develop due to?
A. Antidiuretic hormone
B. Atriopeptide
C. Glucagon
D. Insulin
E. Cortisone
3. A patient, aged 80, complains of increased appetite, thirst, elevated urination, and worsening
of general condition after the taking some sweet food. What disease is it?
A. Hypercortisolism
B. Hyperthyroidism
C. Hypothyroidism
D. Diabetes insipidus
E. Diabetes mellitus
4. A man, aged 38, is under the course of treatment for schizophrenia at in-patient department.
Contents of glucose, ketone bodies, and urea in his blood are normal. Shock therapy with regular
injections of insulin has led to development of insular coma, and after that state of patient
becomes better. What is the most possible reason for insular coma?
A. Glucosuria
B. Dehydration of tissues
C. Hypoglycemia
D. Metabolic acidosis
E. Ketonemia
44
5. In worker at polar station, who has been working there for a long time, hemorrhage form gums
occur, his teeth sway and pull out. What vitamin deficiency leads to these changes?
A. Tocopherol
B. Ergocalciferol
C. Ascorbic acid
D. Folic acid
E. Nicotinic acid
6. A patient, aged 50, complains of increased appetite, thirst, and loss of body weight, weakness.
At laboratory examination rise of amount of glucose in his blood revealed. What type of cells is
injured in case of this disease development?
A. Lipotropocytes
B. Thyrocytes
C. B-cells of Langerhans islets
D. A-cells of Langerhans islets
E. Pancreatocytes.
7. In 18-years-old patient, while laboratory examining presence of glucose in urine and normal
concentration of glucose in blood plasma were revealed. What disorder is the most possible
cause of these changes?
A. Disorders of glomerular filtration
B. Disorders of tubular secretion
C. Disorders of glucocorticoids secretion
D. Disorders of insulin secretion
E. Disorders of tubular reabsorption
8. Dyspepsia and vomiting are observed in a newborn after feeding with milk. These phenomena
disappear after feeding with glucose solution. What enzyme that takes part in carbohydrate
digestion is deficient in case of these changes development?
A. Amylase
B. Lactase
C. Maltase
D. Isomaltase
E. Saccharase
9. In patient painfulness along large nervous trunks and increase of pyruvate in blood are
revealed. What vitamin deficiency may lead to these changes?
A. Pantothenic acid
B. Nicotinic acid (PP)
C. Biotin
D. Riboflavin (B2)

E. Thiamin (B1)
10. A newborn was admitted to the emergency department with following symptoms: vomiting,
diarrhea, disorders of growth and development, cataract, and mental retardation. Galactosemia
was diagnosed. What enzyme deficiency takes place in this case?
A. Glucose-1-phosphate uridiltransferase
B. Glucokinase
C. Glucose-6-phosphate dehydrogenase
D. UDP glucose pyrophosphorilase
E. UDP glucose-4-epimerase
11. Flatulence, bowel spasms, abdominal pain and diarrhea often develop in some people after
taking milk. These symptoms arise in 1 - 4 hours after intake only one glass of milk. What
component of milk these symptoms develop due to?
A. Galactose
B. Lactose
45
C. Maltose
D. Saccharose
E. Fructose
12. Newborn has been refusing food, having vomiting and diarrhea, and some time later its
crystalline lens become opaque. At examination of newborn: glucose in blood – 8.5 mmol/L and
in urine – 1%. What is the most possible diagnosis?
A. Galactosemia
B. Phenylketonuria
C. Tyrosinosis
D. Cystinuria
E. Alkaptonuria
13. In woman, aged 45, without symptoms of diabetes mellitus, content of glucose in blood on
an empty stomach reaches 7.5 mmol/L. What test is necessary to be performed?
A. Determination of residual nitrogen in blood
B. Determination of glucose in blood on an empty stomach
C. Determination of tolerance to glucose
D. Determination of ketone bodies in urine
E. Determination of glycosylated hemoglobin
14. In man, aged 60, who is 170 cm tall and 110 kg weight, content of glucose in blood is 6.8–
7.0 mmol/L. Content of insulin in his blood is normal. Tolerance to glucose is decreased, as well
as number of insulin receptors, in this patient. What signs allow us to evaluate this diabetes
mellitus as non-insulin dependent?
A. Decreased tolerance to carbohydrates
B. Recurrent hyperglycemia
C. Patient’s age
D. Normal content of insulin in blood
E. Obesity
15. In a patient, suffered from frequent hemorrhages from internal organs and mucous
membranes, proline and lysine were found in structure of collagen fibers. What vitamin
deficiency contributes disorders of these amino acids hydroxylation?
A. Vitamin C
B. Vitamin E
C. Vitamin K
D. Vitamin A
E. Vitamin B1
16. In patients who suffered from alcoholism B1 hypovitaminosis is often observed, as a
consequence of nutrition disturbances. Symptoms of vitamin B1 deficiency are disorders of
nervous system, psychoses, and amnesia. Why cells of nervous tissue are particularly susceptible
to thiamine deficiency?
A. Aerobic decay of glucose is broken
B. Liplysis in adipose tissue is increased
C. Oxidation of fatty acids is broken
D. Glycolysis is intensified
E. Glycolysis is decreased
17. According to results of gastric juice analysis following traits were revealed: common acidity
– 24 mmol/L, free hydrochloric acid – 1.5 mmol/L, content of gastric mucoprotein is decreased.
What vitamin deficiency is observed in organism?
A. Cobalamin
B. Folic acid
C. Pantothenic acid
D. Nicotinamide
E. Bioflavonoids
46
18. A 24-years-old woman complains of dryness in mouth and loss of weight in spite of good
appetite. At examination of the patient: height – 162 cm, weight – 65 kg, content of glucose in
blood – 8.3 mmol/L, and presence of glucose in urine. What disease does these symptoms
characteristic for?
A. Diabetes mellitus
B. Steroid diabetes
C. Diabetes insipidus
D. Alimentary glucosuria
E. Renal diabetes
19. A man, who have been suffering from diabetes mellitus for a long time, was admitted to the
hospital because of rapid worsening of his condition: general malaise, polyuria, polydipsia,
nausea and vomiting, confusion, sleepiness. Kussmaul respiration and scent of acetone from
mouth were observed in this patient. In his urine high contents of glucose and acetone bodies
were found. What is the reason for worsening of patient’s condition?
A. Diabetic ketoacidosis
B. Gas acidosis
C. Heart failure
D. Renal failure
E. Hypoglycemic coma
20. Treatment of the child for rachitis using vitamin D3 was not efficient. What is the most
possible reason for ineffectiveness of treatment?
A. Disorders of hydroxylation of vitamin D3
B. Deficit of lipids in food
C. Disorders of including vitamin D3 into enzyme
D. Intensified using vitamin D3 by intestine microflora
E. Disorders of transport of vitamin D3 with blood plasma proteins
21. Content of glucose in patient’s blood is: on an empty stomach – 5.65 mmol/l, in one hour
after taking sugar – 8.55 mmol/l, and in two hours after taking sugar – 4.95mmol/l. These signs
are characteristic for:
A. Healthy person
B. Person suffered from hidden diabetes mellitus
C. Person suffered from non-insulin-dependent diabetes mellitus
D. Person suffered from insulin-dependent diabetes mellitus
E. Person suffered from thyrotoxicosis
22. Patient, aged 26, who suffered from hypoglycemic coma resulted from insulin overdosage,
was intravenously infused with 20% solution of glucose. After this manipulation patient’s
condition improved. What process helps glucose enter the cell?
A. Osmotic transport
B. Pinocytosis
C. Active transport
D. Secretion
E. Phagocytosis
23. A 40-years-old man is suffering from diabetes mellitus. After he has endured tonsillitis,
reinforcement of thirst, nausea, vomiting, abdominal pain, and sleepiness develop in him.
Patient’s BP is 80/45 mmHg, pulse rate 125 bpm, and his skin is dry. Content of glucose in blood
is 28 mmol/L. What complication of diabetes mellitus appears in this patient?
A. Lactic acidosis
B. Diabetic ketoacidosis
C. Hepatic coma
D. Hyperosmolar coma
E. Hypoglycemic coma

47
24. After the break of diet (taking the easy for assimilation carbohydrates) in a woman, who has
been suffering from diabetes mellitus for a long time, general malaise and increase of blood
pressure gradually develop and hallucinations and cramps appear. Woman has dry skin and
distinct signs of dehydration. What is the reason for worsening of patient’s condition?
A. Hypoglycemic coma
B. Hyperosmolar hyperglycemic coma
C. Diabetic ketoacidosis
D. Heart failure
E. Respiratory failure
25. During the experiment rat was injected with 5% alloxan solution in dose 200 mg per kg of rat
weight. What kind of pathology arises in this case?
A. Arterial hypertension
B. Acute renal failure
C. Diabetes insipidus
D. Diabetes mellitus
E. Hepatic failure
26 One-year infant lags in mental development from infants of the same age. The infant has
vomiting, cramps, and loss of consciousness in the mornings. What enzyme deficiency these
changes are connected to?
A. Phosphorylase
B. Arginase
C. Glycogen syntase
D. Saccharase
E. Lactase
27. In patient with constant hypoglycemia blood analysis does not change after injection of
adrenalin. A doctor supposes hepatic disorder. What function disorder it is?
A. Cholesterol formation
B. Excretion
C. Glycolysis
D. Ketogenesis
E. Glycogen deposition
28. A woman, aged 58, was admitted to the hospital in severe condition. She has confused
consciousness; dry skin, hollow eyes, cyanosis, and scent of rotten apples from her mouth. At
laboratory examination of her: glucose in blood – 15.1 mmol/L, glucose in urine – 3.5%. What is
the most possible reason for this condition?
A. Hypoglycemic coma
B. Hyperglycemic coma
C. Hypovolemic coma
D. Uremic coma
E. Anaphylactic shock
29. A doctor reveals in child symmetric roughness on cheeks, diarrhea, and disorders of neural
activity. What nutrition factors deficit underlies this condition?
A. Methionine, lipoic acid
B. Lysine, ascorbic acid
C. Nicotinic acid, tryptophan
D. Threonine, pantothenic acid
E. Phenylalanine, pangamic acid
30. The most of participants of Magellan expedition to America died of avitaminosis. This
disease displays as general malaise, subcutaneous hemorrhage, pulling teeth out, hemorrhage
from gums. What is the name for this avitaminosis?
A. Scurvy
B. Pellagra
48
C. Addison-Birmer’s anaemia
D. Polyneuritis (beriberi)
E. Rachitis
31. In a woman, of 52 years old and of 125 kg weight, diabetes mellitus develops. It happens due
to:
A. Decrease of number of insulin receptors
B. High-affinity binding insulin to synalbumin
C. Reduced cell susceptibility to insulin
D. Increase activity of insulinase
E. Broken insulin synthesis
32. Unconscious patient was admitted to the hospital. He has Kussmaul respiration, blood
pressure 80/50 mmHg, and acetone scent from his mouth. What substances accumulation in
organism may lead to these disturbances?
A. Complex carbohydrates
B. Carbonic acid
C. Lactic acid
D. Modified lipoproteins
E. Ketone bodies
33. In patient suffered from diabetes mellitus metabolic acidosis develops due to accumulation of
ketone bodies (beta-oxybutyric acid and acetoacetic acid). At this condition pH of arterial blood
is:
A. 7.40
B. 7.48
C. 7.56
D. 7.32
E. 7.66
34. A patient, 56 years old woman, who have been suffering from diabetes mellitus for 6 years,
complains of compressing pain behind her sternum. What mechanism of heart affection is the
most possible in this case?
A. Myocardial dystrophy
B. Microangiopathy of myocardial vessels
C. Macroangiopathy of coronary vessels
D. Myocarditis
E. Vegetative neuropathy of heart
35. Content of glucose in patient’s blood is: on an empty stomach – 4.52 mmol/l, in one hour
after taking sugar – 6.23 mmol/l, and in two hours after taking sugar – 2.56 mmol/l. These signs
are characteristic for:
A. Healthy person
B. Person suffered from insulinoma
C. Person suffered from hidden diabetes mellitus
D. Person suffered from insulin-dependent diabetes mellitus
E. Person suffered from thyrotoxicosis
36. What complication may develop when treating diabetic ketoacidosis with large doses of
insulin?
A. Arterial hypertension
B. Leukocytosis
C. Arterial hypotension
D. Hypoglycemia
E. Hyperkalemia
37. A female patient, aged 24, complains of dryness in her mouth, loss of weight despite good
appetite. Her height is 157 cm her weight is 72 kg. What analysis we have to perform at fist in
this patient?
49
A. Urinalysis by Zemnitsky
B. Determination of glucose content in 24-hour urine quantity
C. General urinalysis
D. Determination of protein fractions in blood serum
E. Coagulogram
38. A patient D., 40-years-old woman, was admitted to the hospital with complaints of weakness,
giddiness, hunger, cold sweat, and cramps. At examination of the patient: distension of pupils of
the eyes, weakening of respiration, and BP is 90/50 mmHg. Biochemical analysis of her blood
shows: general bilirubin is 16.0 mcmol/L, urea is 4.7
mmol/L, creatinine is 98 mcmol/L, and glucose is 2.0 mmol/L. What kind of coma may develop
in this patient?
A. Hyperglycemic
B. Hepatic
C. Renal
D. Hypoglycemic
E. At adrenal glands deficiency
39. The woman B., aged 45, was admitted to the emergency department from a street. At
examination of patient following symptoms was revealed: loss of consciousness, loss of reflexes,
absence of reflexes from pupil of the eye and sclera, Kussmaul respiration, BP is 70/40 mmHg,
and body temperature is 35oC. Biochemical analysis of patient’s blood displays: general
bilirubin is 16.0 mcmol/L, urea is 3.6 mmol/L, creatinine is 108 mcmol/L, and glucose is 22
mmol/L. What kind of coma has developed in this patient?
A. Hypoglycemic
B. Hepatic
C. Renal
D. At adrenal glands deficiency
E. Hyperglycemic
40. Diabetes mellitus develops in animals after injection to them some alloxan. What is the main
mechanism of this type of diabetes mellitus?
A. Selective damage of lambda-cells of pancreatic islets
B. Damage of beta- and lambda-cells of pancreatic islets
C. Formation of antibodies to insulin
D. Selective damage of beta-cells of pancreatic islets
E. Gluconeogenesis activation
41. A patient address to a doctor with complaints of constant thirst. Hyperglycemia, polyuria and
increased content of 17-ketosteroids in urine were revealed. What is the most probable disease in
this case?
A. Addison’s disease
B. Myxedema
C. Glycogenosis of I type
D. Insulin dependent diabetes mellitus
E. Steroid diabetes
42. In 62-years-old woman cataract (crystalline lens dimness) develops at the diabetes mellitus
background. What process enhancement due to diabetes mellitus is the cause of cataract?
A. Ketogenesis
B. Lipogenesis
C. Proteolysis
D. Glycosilation of proteins
E. Gluconeogenesis
43. A 19-years-old patient has suffered from diabetes mellitus since he was 8. He took cure
irregularly. He was admitted to the hospital in connection to diabetes ketoacidosis development.
What kind of respiration is the most possible in this condition?
50
A. Kussmaul respiration
B. Biot’s respiration
C. Chane-Stocks respiration
D. Inspiratory breathlessness
E. Expiratory breathlessness
44. A patient aged 60 has been suffering from diabetes mellitus for 18 years. He complains of
cold of his lower extremities and intermittent lameness during last years. What is the mechanism
of indicated symptoms development?
A. Macroangiopathy of lower extremities
B. Hyperketonemia
C. Neuropathy
D. Disturbances of metabolism of muscles
E. Hyperglycemia
45. An adolescent aged 17 addressed to a physician in connection with enter a college. He has no
complaints. His height is 178 cm; his weight is 96 kg without essential changes during a year. He
has even distribution of subcutaneous adipose tissue. His pulse rate is 82 bpm; his BP is 115/80
mmHg. At laboratory examination of a youth: content of glucose in blood is 8.2 mmol/L;
glucosuria – 4.6 g/L. What type of diabetes mellitus is the most probable in this patient?
A. Non-insulin-dependent diabetes mellitus with obesity
B. Insulin-dependent diabetes mellitus
C. Symptomatic diabetes mellitus is due to encephalic syndrome
D. Steroid diabetes mellitus is due to Cushing’s syndrome
E. Symptomatic diabetes mellitus is due to acromegaly
46. Signs of fatty dystrophy of liver are revealed in 38-years-old female patient who has been
suffering from diabetes mellitus for a long time. What factor deficiency is leading in this state
development?
A. Lipocain
B. Lecithin
C. Insulin
D. Glycogen
E. Acetyl-CoA
47. Hyperglycemia and decreased number of insulin receptors on the lipocytes are established in
mice with hereditary obesity. What is the primary mechanism of lipogenesis intensification in
these animals?
A. Hyperinsulinemia
B. Hypoinsulinemia
C. Hyperfunction of lipocytes
D. Decreased tolerance to glucose
E. Increased fat deposition
48. A patient with diabetes mellitus did not take the dose of insulin in time. Thus gyperglycemic
coma developed (content of glucose in patient’s blood is 50 mmol/L). What is the leading factor
for such coma development?
A. Hyperosmolarity of blood plasma
B. Hypokalemia
C. Hypoxia
D. Hyponatremia
E. Acidosis
49. What is the mast important mechanism of hypoglycemic coma development?
A. Carbohydrate starvation of brain
B. Reinforcement of glycogenilysis
C. Oppression of glycogenesis
D. Oppression of gluconeogenesis
51
E. Intensification of ketogenesis
50. What is the reason for glucosuria appearance under diabetes mellitus?
A. Hyperglycemia
B. Increase of renal threshold
C. Rise of hexokinase activity
D. Increase of glomerular membrane permeability
E. Polyuria
51. In a patient with diabetes mellitus loss of consciousness and cramps were observed after
insulin injection. What is a possible result of blood test for glucose in this case?
A. 5.5 mmol/L
B. 3.3 mmol/L
C. 8.0 mmol/L
D. 10 mmol/L
E. 2.5 mmol/L
52. A patient was admitted to the hospital in connection to osteomyelitis of lower jaw. When
laboratory examining of the patient glucose was found in her urine. Content of glucose in her
blood is normal. What is the reason for glucosuria?
A. Insufficiency of tubular enzyme systems
B. Increase of glomerular filtration
C. Enhancement of tubular secretion
D. Decrease of renal concentration function
E. Rise of osmotic pressure in tubules
53. In a patient, who has poisoning with flodzine, glucosuria was revealed; concentration of
glucose in patient’s blood is 5.6 mmol/L. What is the mechanism of glucosuria in this case?
A. Disturbances of tubular reabsorption
B. Reduction of glomerular filtration
C. Increase of glomerular filtration
D. Disorders of tubular secretion
E. Rise of oncotic pressure of blood

Set 11 checked
1. At examination of patient following were found: hyperglycemia, ketonuria, polyuria,
hyperstenuria, and glucosuria. What kind of acid-base balance disturbances occurs in this case?
A. Gas alkalosis
B. Non-gas alkalosis
C. Metabolic alkalosis
D. Metabolic acidosis
E. Gas alkalosis
2. Acidosis develops in case of severe form of diabetes mellitus. What buffer system components
change at first?
A. Bicarbonate
B. Phosphate
C. Hemoglobin
D. Oxihemoglobin
E. Protein
3. Prolonged convulsions occur in patient suffered from epilepsy. After that following data of
laboratory analysis of this patient were received: pH – 7.14, pCO2 – 45 mmHg, HCO3- – 14
mmol/l, Na+ - 140 mmol/l, Cl- - 98 mmol/l. What kind of acid-base balance disturbances occurs
in this patient?
A. Metabolic ketoacidosis
B. Metabolic lactoacidosis
52
C. Respiratory alkalosis
D. Metabolic alkalosis
E. There are no disorders of acid-base balance
4. While ascending to mountain in alpinist excitation developed that was replaced with headache,
giddiness, breathlessness, and after that apnea occurred. What kind of acidbase balance
disturbances occurs in this case?
A. Non-gas acidosis
B. Excretory acidosis
C. Gas alkalosis
D. Non-gas alkalosis
E. Gas acidosis
5. A patient suffered from chronic glomerulonephritis has increasing general malaise,
tachycardia with recurrent arrhythmia, confusion, and sleepiness. What kind of acidbase balance
disturbance accompanies uremic coma?
A.Non-gas excretory acidosis
B. Non-gas excretory alkalosis
C. Gas acidosis
D. Gas alkalosis
E. Respiratory alkalosis
6. Pregnant woman has toxicosis accompanied by vomiting of 24 hours duration. After that
tetany cramps and dehydration develop. What kind of shift of acid-base balance leads to
described changes?
A. Gas alkalosis
B. Gas acidosis
C. Non-gas metabolic acidosis
D. Non-gas metabolic alkalosis
E.Non-gas excretory alkalosis
7.A patient suffered from diabetes mellitus was admitted to the hospital because of worsening of
his condition. He has general malaise, polyuria, lethargy, and sleepiness. Kussmaul respiration,
heart arrhythmia, and acetone scent in expired air are noticed in this patient. What kind of shift
of acid-base balance contributes these symptoms?
A. Gas alkalosis
B. Gas acidosis
C. Non-gas metabolic alkalosis
D. Non-gas metabolic acidosis
E. Non-gas excretory alkalosis
8. A group of alpinists was undergone blood analysis in mountains at height 3000 meters.
Following was revealed: decrease of HCO3 - down to 15 mmol/l (norm is 22-26 mmol/l). What
is the mechanism of decrease of HCO3- in the blood?
A. Decrease of reabsorption of bicarbonate in kidneys
B. Hyperventilation
C. Intensification of acidogenesis
D. Hypoventilation
E. Reduction of ammoniogenesis
9. pH of blood of patient suffered from diabetes mellitus sets to 7.3. What component of buffer
system determination is used to diagnose disorders of acid-base balance?
A. Bicarbonate
B. Oxihemoglobin
C. Phosphate
D. Hemoglobin
E. Protein

53
10. Buffer capacity of blood decreases in worker as a result of exhausting muscle work. What
acid substance income to the blood this may be explained?
A. Alpha-ketoglutaric acid
B. 3-phosphoglycerate
C. Lactic acid
D. Pyruvate
E. 1,3-biphosphoglycerate
11. Repeated vomiting occurs in patient suffered from pylorostenosis which is accompanied by
loss of chloride ions from the organism and development of non-gas alkalosis. What conditions
these changes of acid-base balance may result from?
A. Hyperchloridemia
B. Hyponatremia
C. Hypokalemia
D. Hypernatremia
E. Hyperhposphatemia
12. 48-years-old patient with diabetes mellitus was admitted to the hospital in severe pre-coma
state. When examining of acid-base balance metabolic acidosis was revealed. Patient was treated
with complex therapy including insulin intramuscular injections and sodium bicarbonate solution
intravenous infusion. What is the main possible mechanism of found changes development?
A. Disorders of O2 using in cells
B. Disorders of buffer systems of blood
C. Decrease of CO2 removing
D. Excretion of alkali elements with urine
E. Formation of products of incomplete oxidation
13. Patient has disturbances of airways passage at the level of small and medium bronchi. What
kind of acid-base disorders may develop in this patient?
A. Respiratory alkalosis
B. Metabolic alkalosis
C. Respiratory acidosis
D. Metabolic acidosis
E. Acid-base balance does not change
14. In patient suffered from diabetes mellitus coma has developed due to disorders of acid-base
balance. What kind of acid-base disorders develops in this case?
A. Exogenous acidosis
B. Respiratory acidosis
C. Metabolic acidosis
D. Gas alkalosis
E. Non-gas alkalosis
15. A woman has distinct hypersalivation syndrome. Patient has to remove saliva from her
mouth with napkin because of hard pain when she is swallowing. What kind of acid-base
disorders may develop in this patient in some time?
A. Non-gas excretory acidosis
B. Gas alkalosis
C. Metabolic acidosis
D. Non-gas excretory alkalosis
E. Gas acidosis
16. During bronchial asthma attack patient developed gas acidosis (hypercapnia). What buffer
system of blood plays the main role in compensation of this condition?
A. Bicarbonate
B. Hemoglobin
C. Phosphate
D. Protein
54
E. Ammoniogenesis
17. At examination of a patient hyperglycemia, ketonuria, polyuria, hyperstenuria, and
glucosuria were revealed. What type of acid-base balance disorder takes place in this case?
A. Metabolic acidosis
B. Gas acidosis
C. Metabolic alkalosis
D. Non-gas alkalosis
E. Gas alkalosis
18. A 65-years-old patient with multiple fractures of ribs was admitted to the hospital. What type
of acid-base balance disorder may develop in him?
A. Gas acidosis
B. Gas alkalosis
C. Non-gas acidosis
D. Non-gas alkalosis
E. There are no disorders of acid-base balance
19. A pregnant woman has toxicosis, which accompanied by prolonged vomiting. Data of her
biochemical analyses show: pH of blood – 7.38, pCO2 of arterial blood – 46 mmHg, SB – 17
mmol/L, BE – (+ 6 mmol/L). What type of acid-base balance disorder takes place in this case?
A. Compensated non-gas alkalosis
B. Decompensated non-gas alkalosis
C. Compensated non-gas acidosis
D. Decompensated non-gas acidosis
E. Compensated gas alkalosis
20. A 58-years-old female patient was admitted to the hospital in severe state. Data ofher
biochemical analyses show: pH of blood – 7.33, pCO2 of arterial blood – 36 mmHg, SB – 17
mmol/L, BE – (+ 6 mmol/L). What type of acid-base balance disorder takes place in this case?
A. Compensated non-gas alkalosis
B. Decompensated non-gas alkalosis
C. Compensated non-gas acidosis
D. Decompensated non-gas acidosis
E. Compensated gas alkalosis
21. What is the reason for gas alkalosis?
A. Pulmonary hyperventilation
B. Loss of gastric juice
C. Loss of intestine juice
D. Pulmonary hypoventilation
E. Hyperaldosteronism
22. What kind of acid-base balance disturbances may be observed in case of diabetes mellitus?
A. Non-gas acidosis
B. Gas acidosis
C. Gas alkalosis
D. Non-gas alkalosis
E. Excretory acidosis

Set 12 checked
1. It is characteristically for cholera to lose large quantity of water and sodium ions from the
organism. The basis of biochemical action of cholera toxin is:
A. Oxidation of aldosterone in the cortex of adrenal glands
B. Activation of adenilat cyclase in enterocytes of small intestine
C. Intensification of renin secretion by the cells of juxtaglomerular apparatus
55
D. Decrease of synthesis of antidiuretic hormone in hypothalamus
E. Activation of synthesis of atrial natriuretic hormone
2. Wasp has stung a woman. Oedema and hyperemia develop at woman’s left cheek (site of the
bite). What mechanism of oedema development is primary in this case?
A. Decrease of osmotic pressure of blood
B. Impediment for lymphatic drainage
C. Increase of oncotic pressure of tissue fluid
D. Increase of capillary permeability
E. Increase of hydrostatic pressure of blood in capillaries
3. In worker of fusing workshop thirst develops due to enhanced sweating. He drinks a lot of
water without salt. What kind of water-salt balance disturbance the most possibly develops in
this case?
A. Hyperosmolar hyperhydration
B. Hyperosmolar hypohydration
C. Hypoosmolar hyperhydration
D. Hypoosmolar hypohydration
E. Isoosmolar hypohydration
4. A boy, aged 9, with oedemas was admitted to the hospital. What proteins content in blood is
decreased in this case?
A. Albumins
B. Protamines
C. Globulins
D. Hemoglobin
E. Hystons
5. Worker of fusing workshop, man of 23 years old and of 60 kg weight, was admitted to the
emergency department. Examination of water-salt exchange in this patient displays: content of
general water is 33 liters (55% of body weight), extracellular sector constitutes 28.6% of body
weight (17.2 liters), intravascular fluid constitutes 4% of body weight (2.4 liters), and
intracellular sector constitutes 26.3% of body weight (15.8 liters). Osmotic pressure of patient’s
blood is 340 mosm/l; content of sodium in it - 160 mmol/l. Patient’s urination is 0.4 liters per 24
hours. Determine the type of dyshydration.
A. Hyperosmolar hyperhydration
B. Hyperosmolar hypohydration
C. There is no dyshydration
D. Isoosmolar hyperhydration
E. Hypoosmolar hypohydration
6. A patient with severe nephropathy accompanied by severe oedema syndrome that develops as
complication of bronchiectasis. Laboratory examination of this patient displays abundant
proteinuria, cylinderuria, distinct decrease of protein content in blood serum, hyperlipidemia,
hypokalemia, and other pathological changes. What is the most important link in development of
oedemas in this patient?
A. Decrease of oncotic pressure of blood
B. Increase of osmotic pressure of interstitial fluid
C. Increase of hydrostatic pressure of blood
D. Blockade of lymphatic drainage
E. Increase of microvessel permeability
7. A patient has increased osmolarity of urine and decreased urination due to intensive sweating
and dehydration. What hormone secretion changes provide compensatory retention of water at
first?
A. Antidiuretic hormone
B. Aldosterone
C. Corticosterone
56
D. Thyroxin
E. Insulin
8. Content of sodium in patient’s blood serum is 100 mmol/l. What does this condition may
manifest in?
A. Edemas
B. Arrhythmias
C. Dehydration
D. Heart arrest
E. Tachycardia
9. In a patient, aged 44, thirst develops after burns. What receptors generate impulses that
underlie thirst development in this case?
A. Osmoreceptors
B. Pain receptors
C. Thermal receptors
D. Tactile receptors
E. Chemoreceptors
10. Isoosmolar hypohydration has been formed in a patient due to severe diarrhea. What
symptoms are characteristic for this disturbance of water exchange?
A. Edematous syndrome
B. Decrease of content of water inside cells
C. Hypovolemic shock
D. Polyuria
E. Arterial hypertension
11. In patient with affection of kidneys hypoosmolar hyperhydration (water poisoning) has
developed. What is the main pathogenic factor of this syndrome?
A. Anuria
B. Hypoaldosteronism
C. Polyuria
D. Hypoproteinemia
E. Increase of microvessel permeability
12. Hyperosmolar hypohydration has been formed while prolonged water starvation. Which of
following manifestations are typical for this condition?
A. Arterial hypertension
B. Hypoisostenuria
C. Hypothermia
D. Increased salivation
E. Cramps and hallucinations
13. In patient suffered from severe chronic glomerulonephritis retention of isoosmolar fluid in
organism and distinct edematous syndrome occur. What is the major factor of edema
development in case of glomerulonephritis?
A. Hyperproteinemia
B. Secondary aldoseronism
C. Hypoproteinemia
D. Hypoaldosteronism
E. Arterial hypertension
14. In patients with myeloma content of proteins in blood plasma is increased up to 200 g/l. This
leads to redistribution of water between intracellular, interstitial, and intravascular spaces. What
direction does water mainly move at in this case?
A. From interstitium to blood vessels
B. From cells to interstitium
C. From interstitium to cells
D. From blood vessels to interstitium
57
E. From lymphatic vessels to interstitium
15. A patient of 18 years old and of 60 kg weight was admitted to the hospital with signs of
hemic hypoxia resulting from poisoning by nitric compounds. Examination of water exchange of
this patient displays: general water – 64% of body weight, extracellular fluid – 18%,
intravascular fluid – 5%, and intracellular fluid – 46%; osmotic pressure of blood plasma is 250
mosm/l; urination is 0.8 L per 24 hours. Define the type of dyshydration?
A. There is no dyshydration
B. Isoosmolar hyperhydration
C. Isoosmolar hypohydration
D. Hypoosmolar hyperhydration
E. Hypoosmolar hypohydration
16. Edemas at lower extremities occur in a patient suffered from severe heart failure. What is the
leading mechanism of edema development in this case?
A. Centralization of blood circulation
B. Lowering of hydrostatic pressure
C. Secondary hyperaldosteronism
D. Orthostatic increase of venous pressure
E. Hypoproteinemia
17. What is the initial link in formation of heart edemas?
A. Increase of vascular permeability
B. Decrease of minute heart volume
C. Activation of renin-angiotensin system
D. Increase of content of aldosteron in blood
E. Increase of secretion of antidiuretic hormone
18. What is the leading factor of edema development in case of nephrotic syndrome?
A. Increase of hydrostatic pressure in capillaries
B. Increase of vascular permeability
C. Hypoalbuminemia
D. Dynamic lymphatic insufficiency
E. Increase of blood volume
19. Patient suffered from cirrhosis of liver was given with 500 ml of 5% glucose solution with
medicines. What disturbances of water-salt balance may appear in this patient?
A. Hypoosmolar hyperhydration
B. Hyperosmolar hyperhydration
C. Isoosmolar hyperhydration
D. Hypoosmolar hypohydration
E. There is no dyshydrotation
20. Patient has edema of right lower part of face and pulsing pain in tooth, which intensifies
when taking some hot food. Dentist has diagnosed acute pulpitis. What is the leading mechanism
of edema development in this case?
A. Disorders of microcirculation in the focus of inflammation
B. Disorders of trophic function of nervous system
C. Hypoproteinemia
D. Hyperosmia
E. Lymphocytosis
21. Toxic lung edema was modeled in rat using solution of ammonium chloride. What is the
leading mechanism of edema development in this case?
A. Reducing of colloid-osmotic pressure
B. Rising of venous pressure
C. Increase of vascular permeability
D. Disorders of neural and humoral regulation
E. Intensification of lymphatic drainage
58
22. A rat was intravenously injected by 10 ml of 40% glucose solution. In 60 min coma develops
due to hyperosmolar dehydration in this rat. What is the mechanism of edema development in
this case?
A. Loss of water and salts
B. Reduction of vasopessin synthesis
C. Increase of oncotic pressure of extracellular fluid
D. Increase of osmotic pressure of extracellular fluid
E. Disturbance of acid-base balance
23. Patient has extracellular edema of tissues (dimensions of soft tissues of extremities, liver, and
others are enlarged). What parameter of homeostasis decrease do these changes result from?
A. Viscosity
B. pH
C. Hematocrite
D. Oncotic pressure of blood plasma
E. Osmotic pressure of blood plasma
24. At complete starvation (with taking water) generalized edemas develop. What is the leading
pathogenic factor in this case?
A. Increase of oncotic pressure of interstitial fluid
B. Decrease of osmotic pressure of blood plasma
C. Decrease of oncotic pressure of blood plasma
D. Increase of osmotic pressure of interstitial fluid
E. Decrease of hydrostatic pressure of interstitial fluid
25. When treating for dehydration by means of salt-poor fluids at the background of sharply
reduced excretory renal function resulted from tubular necrosis the worsening of general
condition, confused consciousness, convulsive readiness, and brain edema with vomiting
develop. What kind of water-salt exchange disturbances takes place in
this case?
A. Hypoosmolar hyperhydration
B. Isoosmolar hyperhydration
C. Hyperoosmolar hyperohydration
D. Hypoosmolar hypohydration
E. Hyperosmolar hypohydration
26. As it is known, general amount of water in an organism depends on age, body weight, and
sex. Besides, pathogenetic classification of dehydration is important. This classification includes
forms of mineral salt deficiency, forms of water deficiency, and transitional forms. What
reasons, according to this classification, belong to form of mineral salt deficiency?
A. Loss of electrolytes through the stomach
B. Loss of water is due to polyuria
C. Reduced intake of water
D. Loss of water with a bleeding
E. Loss water is due to hyperventilation
27. A patient has uneasiness in the chest and difficult breathing after physical exertion. Some
time later cough with foamy liquid phlegm appears. Significant cyanosis develops in the patient.
What is the leading mechanism for edema development in this case?
A. Hydrodynamic
B. Colloid
C. Membranogenous
D. Lymphogenic
E. Osmotic
28. In inflammation increased vascular permeability and increase of hydrostaticpressure are
observed in microcirculatory vessels. The level of colloid-osmotic pressure of blood does not
significantly changes. There are shift of pH towards acid state, rise of osmotic pressure, and
59
increase in dispersion of proteins in the interstitial fluid. What kind of edema is observed in this
case?
A. Mixed
B. Hydrodynamic
C. Colloid-osmotic
D. Lymphogenic
E. Membranogenous
29. A person, who has been starving for along time, has edemas. What is the main mechanism of
edema development in this case?
A. Decrease of oncotic pressure of the blood
B. Increase of oncotic pressure of tissues
C. Increase of hydrostatic pressure of venous blood
D. Decrease of hydrostatic pressure of tissues
E. Decrease of blood circulating volume
30. In person, who was bitten by bees, edema of the face and the upper extremities developed.
What is the main mechanism of edema development in this case?
A. Increase of vascular permeability
B. Increase of hydrostatic pressure in capillaries
C. Decrease of hydrostatic pressure of tissues
D. Increase of oncotic pressure of tissues
E. Decrease of oncotic pressure of blood
31. Pulmonary hypertension and right-ventricle heart insufficiency with ascites and edemas
develop in a patient suffered from pneumosclerosis. What is the main mechanism of edema
development in this case?
A. Increase of hydrostatic pressure in veins
B. Increase of oncotic pressure of tissues
C. Decrease of oncotic pressure of blood
D. Reduction of heart stroke volume
E. Increase of vascular permeability
32. Which of followings may cause the isoosmolar hypohydration?
A. Acute blood loss
B. Diarrhea
C. Vomiting
D. Perspiration
E. Hyperventilation
33. Where does fluid accumulate predominantly in case of edema?
A. Intercellular space
B. Intracellular space
C. Pleural cavity
D. Abdominal cavity
E. Intravascular space

60

You might also like